Hurst 2

Réussis tes devoirs et examens dès maintenant avec Quizwiz!

A nurse is caring for a client admitted with chronic fatigue and weakness. During the physical assessment, the nurse notes jaundiced sclera, abdominal distension, swelling in the legs and ankles, and bruises in various stages of healing throughout the body. What nursing interventions should the nurse initiate? 1. Measure abdomen 2. Monitor intake and output 3. Obtain daily weight 4. Place on fall precautions 5. Provide three meals per day 6. Dangle legs

1., 2., 3., & 4. Correct: The symptoms presented are indicative of liver disease. Measuring abdominal girth will monitor for accumulating ascitic fluid. Clients with liver disease have fluid volume problems, so daily weight and I&O are indicated. This client is at risk for injury related to chronic fatigue and weakness, so fall prevention is indicated. The client may need help eating if fatigue is severe. 5. Incorrect: Poor tolerance to larger meals may be due to abdominal distension and ascites. Clients should eat smaller, more frequent meals (6/day). The recommended diet is high calorie and low sodium with protein regulated based on liver function. Between meal snacks should be provided. 6. Incorrect: Elevating legs enhances venous return and reduces edema in extremities. Dangling the leg would cause the fluid in the lower extremities to accumulate more.

A home health nurse inspects the home of a client scheduled to be discharged home after receiving care for a cerebrovascular accident with generalized weakness. What safety interventions should the nurse recommend based on findings within the home? 1. Place ramp over the front steps. 2. Move client's bedroom downstairs. 3. Remove throw rugs. 4. Secure furniture so client can use for support. 5. Apply nonskid strips to shower stall.

1., 2., 3., 5. Correct: The client will have difficulty navigating the steps, both outside and inside the home. The client may trip on throw rugs, and shower stalls are slippery when wet. These things, along with the generalized weakness, makes the client more prone to falls. These interventions will promote safety for the client and decrease the risk of falling. 4. Incorrect: Do not have client rely on furniture for support while walking as they may not provide the consistent support needed to prevent falls. The client should use prescribed assistive devices, which are designed to help prevent falls when used properly.

The RN is caring for a client diagnosed with an abdominal aortic aneurysm. Which prescription can the RN delegate to the LPN? 1. Obtain vital signs every 15 minutes. 2. Insert a urinary catheter for hourly urinary outputs. 3. Place a PICC line for fluid management. 4. Provide morphine 1 mg per PCA pump at a 10 minute lockout.

2. Correct. Inserting a urinary catheter is within the scope of practice for the LPN. This task does not include further assessment of the urinary output, which the RN will perform. 1. Incorrect. The UAP can do this task as well as the LPN. In order to be most effective with the nurse's time, this task can be delegated to the UAP. 3. Incorrect. The RN with special training can insert a PICC line. The LPN cannot complete this task. 4. Incorrect. The RN must complete this task. The LPN should not initiate PCA morphine.

A new nurse has a prescription to insert a feeding tube. The new nurse has never performed the procedure, but learned how to do it while in nursing school. What would be the best action by this nurse? 1. Ask to observe another nurse perform the procedure. 2. Look up how to perform the procedure in the policy and procedure manual. 3. Tell the charge nurse that someone else will have to place the feeding tube down the client. 4. Insert the feeding tube as learned in nursing school.

2. Correct. The best action for the nurse to take is to look up how the procedure is done in the agency by looking it up in the policy and procedure manual. The nurse could then discuss the procedure with an experienced nurse and ask the nurse to observe the new nurse while inserting the feeding tube. 1. Incorrect. This is passive and would not benefit the new nurse to strengthen the skills. The best action would be to look up how to do the procedure, discuss with another nurse, and ask that nurse to observe the insertion of the feeding tube. 3. Incorrect. This is not the best option. The new nurse needs to insert the feeding tube in order to become more proficient with this skill. This option will not help the new nurse gain confidence in nursing skills. 4. Incorrect. Although the new nurse should have the basic knowledge of feeding tube insertion, the nurse should follow agency policy and procedure. It is then best to discuss the procedure with another nurse and ask the nurse to observe the feeding tube insertion since this nurse has never performed the skill.

Which client should the nurse place in the room with a 6 year old with glomerulonephritis? 1. Twenty-two month old diagnosed with respiratory syncytial virus (RSV). 2. Four year old with nephrotic syndrome. 3. Three year old admitted with febrile seizures. 4. Two year old who has a fractured tibia.

2. Correct. This child is not infectious and could be placed in the room with the child who has glomerulonephritis. Since the children are close in age, they will adapt well together. 1. Incorrect. Respiratory syncytial virus (RSV) is a common and highly contagious virus that infects the respiratory tract of many children before their second birthday. This client requires contact and droplet precautions and should not be in the room with the client who has glomerulonephritis. 3. Incorrect. Febrile seizures are one of the most common neurologic childhood problems often caused by a fever with a viral infection. Although the underlying infection is not identified, this child with a probable infection should not be placed in the room with the client with glomerulonephritis. 4. Incorrect. Although this child is not infectious, it is not the best option because the child is too young to be in the room with the 6 year old. Place children within the same age group together whenever possible.

The primary healthcare provider has prescribed phenytoin 100 mg intravenous push (IVP) stat for an adult client. What is the least amount of time that the nurse can safely administer this medication? 1. 1 minute 2. 2 minutes 3. 5 minutes 4. 10 minutes Rationale

2. Correct: The rate of IV administration should not exceed 50 mg/min. for adults and 1-3 mg/kg/min (or 50 mg/min, whichever is slower) in pediatric clients because of the risk of severe hypotension and cardiac arrhythmias. So 100 mg can safely be delivered over a period of at least 2 minutes.1. Incorrect: The rate of IV administration should not exceed 50 mg/min. for adults and 1-3 mg/kg/min (or 50 mg/min, whichever is slower) in pediatric clients because of the risk of severe hypotension and cardiac arrhythmias. So 100 mg can safely be delivered over a period of at least 2 minutes. Giving this dose over only one minute could lead to these or other potential harmful effects. 3. Incorrect: The rate of IV administration should not exceed 50 mg/min. for adults and 1-3 mg/kg/min (or 50 mg/min, whichever is slower) in pediatric clients because of the risk of severe hypotension and cardiac arrhythmias. So 100 mg can safely be delivered over a period of at least 2 minutes. Five minutes would be longer than required to be able to safely administer the medication. 4. Incorrect: The rate of IV administration should not exceed 50 mg/min. for adults and 1-3 mg/kg/min (or 50 mg/min, whichever is slower) in pediatric clients because of the risk of severe hypotension and cardiac arrhythmias. So 100 mg can safely be delivered over a period of at least 2 minutes. Ten minutes is much longer than required to be able to safely administer the medication.

The occupational health nurse is caring for an employee after a chemical explosion at the local tire factory. The client reports a foreign body in the right eye. The right eye is watery, and the client reports photophobia. Which nursing action takes priority? 1. Evert eyelid and examine for foreign body. 2. Measure visual acuity. 3. Notify the receiving hospital immediately for transfer of the client. 4. Place an eye shield over eye.

4. Correct: If a foreign body is the result of explosion or blunt or sharp trauma, the eye should be protected from further damage by placing an eye shield over the eye (or if a shield is not available, a paper cup to prevent rubbing of the eye). Then make arrangements to transport the client for emergency care by an ophthalmologist. If movement of the unaffected eye creates movement in the affected eye, it may be necessary to cover the unaffected eye also to prevent further injury to the eye from movement. 1. Incorrect: Everting the eyelid and examining for a foreign body are not measures that should be performed before placement of eye shield. You should never attempt to remove a foreign body, so examination would not be needed at this point. 2. Incorrect: Measuring visual acuity is not a priority and is not performed before placement of eye shield. The goal is to protect the eye from further injury and reduce movement of the eye. The shield will help accomplish this goal. 3. Incorrect: Notifying immediately for transfer should not be done before placement of eye shield. The eye should be protected first to reduce further injury.

A client, admitted in Sickle Cell Crisis, is started on oxygen at 2L/NC and given a narcotic analgesic for pain control. What additional prescription is a priority for the nurse to initiate? 1. A high protein, low fat diet 2. Administration of a thrombolytic, such as streptokinase 3. Implementation of bleeding precautions 4. Administration of IV fluids for hydration

4. Correct: Increasing hydration status via the administration of IV fluids is indicated in sickle cell crisis to increase that volume in the vascular space and subsequently decrease the vaso-occlusion from the sickling effects of the RBCs. The increased volume separates the sickled cells to reduce the clumping together of the cells. 1. Incorrect: While beneficial for many clients, a high protein, low fat diet provides no benefit during the crisis phase of sickle cell disease. Hydration to improve circulation is a priority due to the impairment or obstruction of blood flow caused by the sickled cells clumping together. 2. Incorrect: Thrombolytics are indicated for the lysis of existing clots and do not have a primary role in the treatment or management of sickle cell disease. The issue in sickle cell crisis is not clot formation but rather a clumping together of sickled cells that impairs or blocks circulation. 3. Incorrect: Sickle cell disease is characterized by sickling of RBCs, causing them to clump together and obstruct capillary blood flow, causing ischemia and possible tissue infarction. Increased risk of bleeding is not a concern.

Which client could the charge nurse assign to an LPN/VN? 1. Eight year old in diabetic ketoacidosis (DKA) 2. Six year old in sickle cell crisis 3. Two month old with dehydration 4. Five year old in skeletal traction

4. Correct: The fracture would be most appropriate for an LPN/VN and is within the scope of practice. This LPN/VN would need minimal assistance from the RN. Possibly, the other clients could have intravenous fluid (IVF) needs and medications that would require skill from an RN.1. Incorrect: The child with DKA is in metabolic acidosis. The child is also at risk for other problems such as dehydration and electrolyte disturbances. Therefore, the child will need close observation and the RN's assessment skills.2. Incorrect: IV fluid management is crucial for clients in a sickle cell crisis Assessment of the child's cardiovascular status, tissue perfusion and neuro status are priorities. Pain management is also very important in these clients. Therefore, the child with sickle cell will need close observation and the RN's assessment skills. 3. Incorrect: The baby with dehydration will need close observation and the RN's assessment skills, including monitoring for impending shock. Renal function and electrolyte levels should be monitored closely. The care of the child will likely involve IV fluids.

A client experiencing chest pain is prescribed an intravenous infusion of nitroglycerin. After the infusion is initiated, the occurrence of which symptom would prompt the nurse to discontinue the nitroglycerin? 1. Frontal headache 2. Orthostatic hypotension 3. Decrease in intensity of chest pain 4. Cool, clammy skin

4. Correct: This assessment finding of cool, clammy skin is an indication of decreased cardiac output that could be the result of too much vasodilatation. Cardiac output could continue to decrease if the nitroglycerin is not discontinued. 1. Incorrect: A headache is an expected common side effect of nitroglycerin administration. The headache is treated with medication. 2. Incorrect: A decrease in blood pressure when rising from a supine or sitting position is a common effect of the vasodilatation that occurs with the administration of nitroglycerin. The client should be advised to change positions slowly. 3. Incorrect: The decrease in the intensity of the client's chest pain is the desired outcome of the nitroglycerin administration.

The nurse is caring for a client on the surgical unit. The primary healthcare provider prescribed morphine sulfate 20 mg IM one time dose. The nurse has available: morphine sulfate in a 20 mL vial, labeled 15 mg per mL. How many mL should the nurse administer? Record answer using one decimal place.

15 mg: 1 mL = 20 mg: x mL 15x = 20 x= 1.33 = 1.

A client was admitted to the medical unit with pneumonia 2 days ago. There is a history of drinking 5-6 martinis every night for the past 2 years. Today, the nurse notes that the client is disoriented to time and place and is seeing imaginary spiders on the ceiling. The nurse cannot understand what the client is saying. What is this client most likely experiencing? 1. Wernicke's Encephalopathy 2. Korsakoff's Psychosis 3. Alcohol Withdrawal 4. Alcohol Withdrawal Delirium

4. Correct: Alcohol Withdrawal Delirium usually occurs on the second or third day following cessation of or reduction in prolonged, heavy alcohol use. Symptoms are the same as for delirium: Difficulty sustaining and shifting attention. Extremely distractible; disorganized thinking; rambling, irrelevant, pressured, and incoherent speech; impaired reasoning ability; disoriented to time and place; impairment of recent memory; delusions and hallucinations. 1. Incorrect: Wernicke's Encephalopathy represents the most severe form of thiamine deficiency in alcoholics. Symptoms include paralysis of the ocular muscles, diplopia, ataxia, confusion, somnolence, and stupor. If thiamine replacement therapy is not given, death will ensue. 2. Incorrect: Korsakoff's Psychosis is identified by a syndrome of confusion, personality changes, loss of recent memory, and confabulation (filling in some memory gaps with different life events or created thoughts). It is frequently encountered in clients recovering from Wernicke's encephalopathy. Coordination may be affected, so the client may have difficulty maintaining balance. Treatment is parenteral or oral thiamine replacement. 3. Incorrect: Alcohol withdrawal typically begins 4-12 hours after cessation of or reduction in heavy and prolonged alcohol use. Symptoms include: coarse tremor of hands, tongue, or eyelids; nausea and vomiting; malaise or weakness; tachycardia; sweating; elevated blood pressure; anxiety; depressed mood or irritability; transient hallucinations or delusions; headache; and insomnia.

Which statement made by a client post-thyroidectomy would require further investigation by the nurse? 1. "I have a tingling feeling of my fingers." 2. "It hurts when I move my head." 3. "I feel pressure in my arm when you take my blood pressure." 4. "My legs are weak."

1. Correct. After this procedure the nurse should worry about the possibility of some of the parathyroids being accidentally removed with resulting hypoparathyroidism. Hypoparathyroidism results in hypocalcemia. Signs and symptoms include tingling, burning, or numbness of lips, fingers, and toes. The muscles may become tight and rigid, and seizures can result. 2. Incorrect. Pain is expected here. The incision is at the base of the neck, so movement of the head would increase the pain. 3. Incorrect. The sensation of pressure in the arm is considered normal when the BP is being measured. You worry if you see carpal spasm (+ Trousseau's) which is indicative of neuromuscular excitability caused by hypocalcemia secondary to the inadvertent removal of some of the parathyroids. 4. Incorrect. Weak/flaccid extremities would be seen with hyperparathyroidism. In this case, we are concerned that the parathyroids may have been removed, resulting in hypoparathyroidism. The weakness in the legs is apparently from a different cause. However, the signs of possible hypoparathyroidism would be the priority to investigate.

A client, who only speaks Spanish, is admitted to the surgical unit. What is the best method for the nurse to inform the client about a pre-surgical procedure? 1. Use an audiotape made in Spanish to inform the client of the pre-surgical procedure. 2. Draw pictures of what the client can expect prior to surgery. 3. Facial expressions and gestures can be used to let the client know what to expect. 4. Enlist the help of a Spanish speaking family friend to tell the client what to expect prior to surgery.

1. Correct: Audiotapes made in the language of high volume clients who speak a language other than English is helpful to inform clients about admission procedures, room and unit orientation, and pre-surgical procedures. The tapes are received from sources where reliability of information is provided. This is the most reliable option for providing accurate information. 2. Incorrect: This is not the best option. Some pre-surgical procedure may be difficult to draw or difficult for the client to understand what was drawn. There is no way to know if the client is understanding what the nurse is trying to communicate through the pictures. Client safety could be compromised if decisions are made based on inaccurate perceptions. 3. Incorrect: This is called "Getting by" and may have to be used when the nurse cannot speak the client's language, and there are no interpreters, audiotapes, or written materials available to inform the client in their language. This is not the best option and should be used only if other more reliable means for interpreting are not available. 4. Incorrect: Disadvantages of using ad hoc interpreters include compromising the client's right to privacy and relying on someone without training as an interpreter. Due to lack of training or experience, ad hoc interpreters may leave out important words, add words, or substitute terms that make communication inaccurate. This may have to be done at times if tapes or other reliable means of interpreting are not available. However, this is not the best option.

A client, who only speaks Spanish, is admitted to the surgical unit. What is the best method for the nurse to inform the client about a pre-surgical procedure? 1. Use an audiotape made in Spanish to inform the client of the pre-surgical procedure. 2. Draw pictures of what the client can expect prior to surgery. 3. Facial expressions and gestures can be used to let the client know what to expect. 4. Enlist the help of a Spanish speaking family friend to tell the client what to expect prior to surgery.

1. Correct: Audiotapes made in the language of high volume clients who speak a language other than English is helpful to inform clients about admission procedures, room and unit orientation, and pre-surgical procedures. The tapes are received from sources where reliability of information is provided. This is the most reliable option for providing accurate information. 2. Incorrect: This is not the best option. Some pre-surgical procedure may be difficult to draw or difficult for the client to understand what was drawn. There is no way to know if the client is understanding what the nurse is trying to communicate through the pictures. Client safety could be compromised if decisions are made based on inaccurate perceptions. 3. Incorrect: This is called "Getting by" and may have to be used when the nurse cannot speak the client's language, and there are no interpreters, audiotapes, or written materials available to inform the client in their language. This is not the best option and should be used only if other more reliable means for interpreting are not available. 4. Incorrect: Disadvantages of using ad hoc interpreters include compromising the client's right to privacy and relying on someone without training as an interpreter. Due to lack of training or experience, ad hoc interpreters may leave out important words, add words, or substitute terms that make communication inaccurate. This may have to be done at times if tapes or other reliable means of interpreting are not available. However, this is not the best option.

A client diagnosed with glomerulonephritis presents with generalized malaise, weight gain, generalized edema, and flank pain. The primary healthcare provider prescribes antibiotics and strict bedrest. What is the best explanation to give the client regarding the strict bedrest prescription? 1. Promotes diuresis 2. Prevents injury 3. Promotes rest 4. Stimulates RBC production

1. Correct: Bedrest and the supine position promote diuresis. When the client is supine, there is a gradual shift of fluids away from the legs toward the thorax, abdomen and head. This increased volume causes the right atrium of the heart to stretch and release ANP, which leads to diuresis: renal blood flow increases due to vasodilation, and aldosterone and ADH secretion are inhibited. 2. Incorrect: Bedrest can keep the client from falling and injuring self; however, that is not why it has been prescribed. 3. Incorrect: Promotion of rest is good, but this is not why the primary healthcare provider prescribed it. Simply promoting does not help improve the symptoms listed. The reason the client needs bedrest should focus on relieving the symptoms listed in the stem. 4. Incorrect: No relationship between bedrest and red blood cell production exists.

Two hours after a gastrectomy, a client has pink tinged drainage from the nasogastric (NG) tube, and the tube appears occluded. What is the nurse's initial action at this time? 1. Call the primary healthcare provider. 2. Reposition the client. 3. Increase the suction level. 4. Irrigate the tube.

1. Correct: Do not tamper with fresh surgery tubes. Call the primary healthcare provider for blood draining from the NG tube after gastrectomy. 2. Incorrect: This delays care and does not resolve an occluded NG tube. 3. Incorrect: Increasing the suction level is very dangerous for the client. This could cause hemorrhage in this client. Don't be a killer nurse! Call the primary healthcare provider. 4. Incorrect: Although the healthcare provider may prescribe for the tube to be irrigated later, the healthcare provider should be notified of the presence of blood initially. Irrigating the fresh NG tube in this situation could lead to increased bleeding.

An 82 year old client tells the nurse at the clinic, "I have lived a good, successful life and married my best friend". Which of Erikson's developmental tasks does the nurse recognize that this client has probably accomplished? 1. Ego Integrity versus Despair 2. Generativity versus Stagnation 3. Intimacy versus Isolation 4. Industry versus Inferiority

1. Correct: Ego Integrity versus Despair is the major task of those 65 and over: The developmental task for this age involves the individual reviewing one's life and deriving meaning from both positive and negative events, while achieving a positive sense of self. If the individual considers accomplishments and views self as leading a successful life, a sense of integrity is developed. On the contrary, if life is viewed as unsuccessful without accomplishing life's goals, a sense of despair and hopelessness develops. 2. Incorrect: Generativity versus Stagnation is the major task for 40-64 year olds. To achieve the life goals established for oneself while also considering the welfare of future generations. The primary developmental task during this middle age period is one in which the individual contributes to society as well as helping to guide future generations. A sense of generativity (sense of productivity and accomplishment) often results from such things as raising a family and helping to better the society. In contrast, those individuals not willing to work to better society and those who are egocentric and self-centered often develop a sense of stagnation (dissatisfaction and the lack of productivity). 3. Incorrect: Intimacy versus Isolation is the objective from 20-39 year olds to form an intense, lasting relationship or a commitment to another person. If the individual cannot form the intimate relationships (possibly due to personal needs) a sense of isolation may develop which can lead to feelings of depression. 4. Incorrect: Industry versus Inferiority is the major task for 6-12 year olds in which they attempt to achieve a sense of self confidence by learning, competing, performing successfully, and receiving recognition from significant others, peers, and acquaintances. The child must develop the ability to deal with the demands of learning new social and academic skills, or a sense of inferiority, failure, or incompetence may result.

A client suffers from migraine headaches. What assessment finding would the nurse expect to find during a migraine attack? 1. Unilateral, pulsating pain quality. 2. Bilateral, pressing/tightening pain quality. 3. Ipsilateral nasal congestion and rhinorrhea. 4. Headache occurs after recovering from a headache treated with narcotics.

1. Correct: Migraine headaches have a pulsating pain quality, unilateral location, moderate or severe pain intensity, aggravated by or causing avoidance of routine physical activity (walking, climbing stairs). During headache at least one of the following accompanies the headache: nausea and/or vomiting; photophobia and phonophobia. 2. Incorrect: This is seen in tension headaches. Headaches last 30 minutes to 7 days. Pain is mild or moderate in intensity. It is not aggravated by routine physical activity. Nausea/vomiting, photophobia and phonophobia are not common manifestations with tension headaches. These usually start gradually, often in the middle of the day. 3. Incorrect: This is associated with cluster headaches, which are severe or very severe unilateral orbital, supraorbital and/or temporal pain lasting 15-180 minutes. Symptoms include stabbing pain in one eye with associated rhinorrhea (runny nose) and possible drooping eyelid on the affected side. The headaches tend to occur in "clusters": typically one to three headaches per day (but may be as many as eight) during a cluster period. 4. Incorrect: Overuse of painkillers for headaches, can, ironically, lead to rebound headaches. Culprits include over the counter medications such as aspirin, acetaminophen or ibuprofen, as well as prescription medications. Too much medication can cause the brain to shift into an excited state, triggering more headaches. Also, rebound headaches are a symptom of withdrawal as the level of medicine drops in the bloodstream. Rebound headaches may have associated issues such as difficulty concentrating, irritability and restlessness but does not typically include photophobia or visual disturbances as seen with migraines.

The nurse evaluates the effectiveness of discharge teaching for a client with type I diabetes mellitus. Which statement by the client would indicate to the nurse that teaching has been effective? 1. "Exercising regularly will decrease my insulin need." 2. "I will need to decrease my insulin dose when I develop an infection." 3. "I need to lose weight since obesity decreases insulin resistance." 4. "Increased stress levels will cause the glucose level in my blood to go down."

1. Correct: Regular exercise decreases the need for insulin. Regular exercise reduces insulin resistance and permits increased glucose uptake by cells. This serves to lower insulin levels and reduce hepatic production of glucose. 2. Incorrect: When an infection occurs, blood sugar increases. The normal response to infection is to increase available glucose to assist in combating the infection. This will increase the requirement for insulin, not decrease it. 3. Incorrect: Obesity increases not decreases insulin resistance, so the cells do not respond normally (are resistant) to insulin. Maintaining a healthy weight with exercise and diet can result in less need for insulin (less resistance to insulin) and less problems in individuals with type 2 diabetes. 4. Incorrect: Emotional upset and undue stress results in increased circulating catecholamines. This will increase the blood glucose levels and increase the requirement for insulin.

The emergency department nurse is assessing a client who presents with severe epigastric pain. The client reports that three rolls of calcium carbonate were consumed in the past eight hours to treat the indigestion. Which blood gas report does the nurse associate with this situation? 1. pH - 7.49, pCO2 - 40, HCO3 - 30 2. pH - 7.32, pCO2 - 48, HCO3 - 20 3. pH - 7.38, pCO2 - 52, HCO3 - 32 4. pH - 7.29, pCO2 - 54, HCO3 - 26

1. Correct: These ABGs are indicative of metabolic alkalosis. The pH is high, the pCO2 is within normal limits and the bicarb is high (alkalosis). So, the excess Tums (calcium carbonate) could have caused metabolic alkalosis. 2. Incorrect: The client is not hypoventilating and would not be in metabolic acidosis because he ate 3 rolls of Tums which is a base. These ABGs are indicative of acidosis. The pH is low (acidosis), the pCO2 is high (acidosis) and the bicarb is low (acidosis). 3. Incorrect: The client is not a long-term COPD client as these ABGs might suggest. These ABGs are indicative of fully compensated respiratory acidosis. The pH is normal. The pCO2 is high (as with chronic retention) and the bicarb is high to help compensate. 4. Incorrect: These ABGs are the result of an acute ventilation problem. They are indicative of respiratory acidosis. The pH is low, the pCO​2 is high, and the bicarb is normal. No compensation has begun at this point.

The nurse is planning care for a pediatric client reporting acute pain with sickle cell crisis? What should the nurse identify as an appropriate goal for this client? 1. Client will report a pain level of less than 2 on a Faces scale. 2. The nurse will administer prescribed pain meds around the clock. 3. Client will only take breakthrough pain medication. 4. Client will use distraction instead of pain medication.

1. Correct: Yes, having a pain level of less than 2 is the best goal for pain and the use of a Faces scale, instead of a numerical scale is age appropriate. Sickle cell crisis is extremely painful, and often times, the pain is not completely relieved during the acute stage. 2. Incorrect: The goal should be client centered. This option is a nursing intervention, not a client goal. 3. Incorrect: We are focusing on client response, not limiting pain meds. The goal of a pain crisis should be aimed at reducing the client's pain. 4. Incorrect: Sickle cell crisis is very painful, and pain medication is needed.

A client asked the nurse what could have caused them to develop right sided heart failure? What would be the best response by the nurse? 1. High blood pressure in the lungs. 2. Long term hypertension. 3. The inability of the mitral valve to close properly. 4. Narrowing of the aorta.

1. Correct: Yes, the right side of the heart pumps to the lungs. When the client has higher pressure in the pulmonary circuit from such things as emphysema, the pulmonary pressure can exceed the systemic pressure. The result is back flow to the right side of the heart and resulting right sided heart failure. 2. Incorrect: No, that's left-sided heart failure. Hypertension increases afterload which can ultimately result in back flow to the left side of the heart and resulting left sided heart failure. 3. Incorrect: Not related to pulmonary hypertension. The mitral valve is located between the left atrium and left ventricle. If mild, there may be little or no obvious symptoms. However, if severe, left sided heart failure may occur. 4. Incorrect: Not related to pulmonary hypertension. Narrowing of the aorta makes it harder to get blood out of the left ventricle (high afterload). The resulting back flow of blood would result in left sided heart failure.

An RN on the general pediatric unit has been reassigned to the spinal/neurology unit. What assignment by the charge nurse would be appropriate for this RN? 1. Child with spina bifida with a previous shunt revision 2. Adolescent who is 4 days post op from a spinal fusion 3. Child with a ventriculoperitoneal shunt one day post-op 4. Child with spinal muscle atrophy who is ventilator assisted 5. Child with cerebral palsy who had a tracheostomy performed this AM

1., & 2. Correct: The child who had a previous shunt revision and the adolescent who is 4 days post spinal fusion will be the most stable and will require the least skill level when compared with the other choices. On a general pediatric unit, the nurse would be familiar with checking for increased ICP, which would be necessary for caring for any client with a previous shunt revision. Immediately postop, the adolescent with spinal fusion would require special turning and lung assessment to prevent and observe for congestion/pneumonia, skills not acquired on a general floor. However, at 4 days postop this client should be ambulating and will not need specialized turning, so the nurse from the general pediatric unit could care for this client. 3. Incorrect: This client is more acute and requires a higher skill level. Nursing care for this child would involve frequent neurologic assessments and monitoring for infection. The child should also be monitored for signs of possible complications including bowel perforation. 4. Incorrect: This client is more acute and requires a higher skill level. The nurse on the general pediatric unit would not be experienced in caring for a child on a ventilator. 5. Incorrect: This client is more acute and requires a higher skill level. A child who is fresh post-op following a tracheostomy is at risk for airway obstruction from thick secretions, mucous plug, blood clot or dislodgement of the tube. Cardio-respiratory arrest can occur from these complications. The child is also at risk for hemorrhage. Nursing care would include frequent suctioning as needed, monitoring for early signs of airway obstruction, and trach care. The nurse from the general pediatric unit may not have the skills required to care for this child who also has cerebral palsy which could complicate the care required.

An adult client has just returned to the nursing care unit following a gastroscopy. Which intervention should the nurse include on the plan of care? 1. Vital sign checks every 15 min x 4 2. Supine position for 6 hours 3. NPO until return of gag reflex 4. Irrigate NG tube every 2 hours 5. Raise four siderails Rationale

1., & 3. Correct: Vital signs post procedure are important to monitor for any post-procedure complications such as bleeding or any signs of respiratory compromise. VS are checked frequently for the first hour post procedure. Any client who has a scope inserted down the throat and has received numbing medication in the back of the throat to depress the gag reflex should be kept NPO until the gag reflex returns.2. Incorrect: Supine position for 6 hours is contraindicated. The HOB should be elevated. In the event the client vomits, he/she is less likely to aspirate with the HOB elevated. Supine position for 6 hours is used after a heart catheterization.4. Incorrect: A client who is going for a gastroscopy procedure cannot have a nasal gastric tube. An NG tube would interfere with the procedure.5. Incorrect: Raising all side rails is a form of restraint. Have the bed in low locked position. Raise three side rails, and have call light within reach.

Which prescriptions would the nurse recognize as being appropriate for the client with shingles? 1. Private room 2. Negative pressure airflow 3. Respirator mask 4. Face Shield 5. Positive pressure room

1., 2. & 3. Correct: According to the current standards of Standard Precautions per the CDC, the client with shingles should be placed on airborne precautions which require the use of a private room with negative pressure airflow and a N-95 respirator mask.4. Incorrect: A face shield is used when there is risk of splashing or spraying of blood or body fluids. This is not required for airborne precautions. 5. Incorrect: Negative pressure is required in order to prevent the airborne infection from spreading outside of the room. Positive pressure is used only in protective environments such as when immunocompromised clients require protection from potential infectious agents outside of the room.

The nurse is caring for a client on the psychiatric unit with a diagnosis of obsessive-compulsive disorder. The client has frequent hand washing rituals. Which nursing interventions would be advisable for this client? 1. Allow time for ritual. 2. Provide positive reinforcement for nonritualistic behavior. 3. Provide a flexible schedule for the client. 4. Remove all soap and water sources from the client's environment. 5. Create a regular schedule for taking client to bathroom.

1., 2. & 5. Correct: Initially meet the client's dependency needs as required to keep anxiety from escalating. Anything that increases the client's anxiety tends to increase the ritualistic behavior. Positive reinforcement for nonritualistic behavior takes the focus off of the ritual. A lack of attention to ritualistic behaviors can help to decrease the ritual. By creating a regular schedule when the client goes to the bathroom, (where the handwashing ritual occurs most frequently) allows the client a structured but limited time for the ritual. This can help give the client a sense of control of the maladaptive behavior until the client can start setting own limits on the behavior and develop more adaptive coping mechanisms. 3. Incorrect: A structured schedule is needed for this client. If the client is allowed to perform the maladaptive behavior whenever desired, the client will not begin to problem solve ways to limit the ritual nor lessen the anxiety associated with the ritualistic behavior. The set schedule helps the client to develop trust with the nurse, knowing that time will be allowed for the behavior until better coping skills are developed. 4. Incorrect: Sudden and complete elimination of all avenues for dependency would create intense anxiety in the client. This increased anxiety would only serve to increase the ritualistic behavior. When time is not allowed for the ritual, the client fears that something bad is happening and the anxiety escalates.

A client diagnosed with serotonin syndrome is admitted to the unit. The nurse is familiar with this adverse reaction to the serotonin reuptake inhibitors. Which symptoms can the nurse expect on assessment? 1. Fever and shivering 2. Agitation 3. Decreased body temperature 4. Constipation 5. Increased heart rate Rationale

1., 2. & 5. Correct: Serotonin syndrome is a group of symptoms that can result from the use of certain serotonin reuptake inhibitors. These symptoms can range from mild to severe and include high body temperature, agitation, increased reflexes, diaphoresis, tremors, dilated pupils and diarrhea. The client is likely to experience shivering with fever. Increased heart rate and blood pressure are also commonly experienced. More severe symptoms, including muscle rigidity and seizures, can occur. If not treated, serotonin syndrome can be fatal. 3. Incorrect: Increased body temperature is expected as is increased diaphoresis.4. Incorrect: Diarrhea, not constipation, is a symptom of serotonin syndrome.

A client with a history of command hallucinations was admitted to the hospital yesterday. What questions are most important for the nurse to ask? 1. "Are you hearing voices today?" 2. "What are the voices saying?" 3. "How are you feeling today?" 4. "Did you have difficulty sleeping last night?" 5. "Are the voices telling you to harm yourself or anyone else?"

1., 2. & 5. Correct: The nurse must assess for hallucinations. The nurse needs to know what the voices are saying to determine the level of threat. The nurse needs to know if the command hallucination exists and whether it involves harming self or others which must be reported. These answers are important to know, as the client has a history of command hallucinations.3. Incorrect: The priority is safety of the client and others on the unit. This question does not get the most essential information related to command hallucinations that may cause the client to engage in behavior that is harmful to self or others. 4. Incorrect: This question does not focus on the problem: command hallucinations. If you assume the worse, you want to know if the voices from the command hallucinations are telling the client to harm self or others.

The nurse is working with a LPN/VN and an unlicensed assistive personnel (UAP). Which clients would be appropriate for the nurse to assign to the LPN/VN? 1. In Bucks traction requiring frequent pain medication. 2. 24 hours post appendectomy. 3. Diagnosed with cholelithiasis and scheduled for surgery in the AM. 4. Admitted 6 hours ago in adrenal insufficiency. 5. Client newly diagnosed with Type 2 diabetes.

1., 2., & 3. Correct These clients are stable and require predictable care that can be done appropriately by the LPN/VN. 4. Incorrect: This client has adrenal insufficiency. Primary adrenal insufficiency occurs when at least 90 percent of the adrenal cortex has been destroyed generally from autoimmune disorders. Secondary adrenal insufficiency can be caused by such things as abrupt stoppage of corticosteroid medications and surgical removal of pituitary tumors. As a result, often both glucocorticoid (cortisol) and mineralocorticoid (aldosterone) hormones may be lacking. This puts the client at risk for fluid volume deficit (FVD) and shock. This requires the higher level assessment skills of the RN. 5. Incorrect: A newly diagnosed client may be unstable and would require assessment, care plan development and teaching for the newly diagnosed diabetic which cannot be performed by the PN.

What information should a nurse include when educating a client regarding buccal administration of a medication? 1. This route allows the medication to get into the blood stream faster than the oral route. 2. Stinging may occur after placing the medication in the cheek. 3. If swallowed, the medication may be inactivated by gastric secretions. 4. The buccal dose of medication will need to be increased from the oral dose. 5. Remove the tablet from buccal area after 15 seconds.

1., 2., & 3. Correct: These are correct statements about buccal administration of medication. Buccal administration involves the medication being placed between the gums and cheek, where it dissolves and becomes absorbed into the bloodstream. The cheek area has many capillaries that allow the medication to be absorbed quickly without having to pass through the digestive system. The degree of stinging experienced depends on the medication being administered. Some effects of certain medications can be lessened by digestive processes. 4. Incorrect: When given by the buccal route, the medication does not go through the digestive system. This means that the medication is not metabolized through the liver, and thus a lower dose can be used. 5. Incorrect: Placement should be maintained until the tablet is dissolved in order to get the dosage and effects desired.

Which immunizations obtained by the age of two would indicate to the pediatric nurse that the child is up-to-date on immunizations? 1. Diptheria-tetanus-pertussis (DTaP). 2. Inactivated polio (IPV). 3. Herpes zoster. 4. Meningococcal 5. Haemophilus influenza type B (Hib).

1., 2., & 5. Correct: By the age of two, the DTaP, IPV, MMR, Hib, varicella, pneumococcal, and rotovirus vaccines should have been received. The nurse should clarify this with the parent. 3. Incorrect: This vaccine is recommended for people 60 years or older whether or not the person has ever had chicken pox and is at risk for developing shingles. Although the vaccine can be given to adults between the ages of 50-59, routine administration is not recommended. 4. Incorrect: The minimum age for administering the meningococcal vaccine is two years of age. The recommended age for administering the meningococcal vaccine is at 11 or 12 years of age, or 13 through 18 years of age if they did not previously receive this vaccine. It is especially important for teens going to college and who are likely to stay in close quarters such as a dorm.

Which interventions should be included in the plan of care for an adult client with constipation? 1. Allow adequate time for defecation. 2. Provide privacy for bowel elimination. 3. Suggest increasing fluid intake (unless contraindicated). 4. Encourage client to increase fiber in the diet. 5. Encourage the client to delay the urge to defecate until after a meal.

1., 2., 3. & 4. Correct: Clients should have ample time for defecation. Rushing the client may lead to a client ignoring the urge. Since clients may be hesitant to have a bowel movement in the presence of others, privacy should be provided. (The nurse may need to stay with weak or disabled clients.) Increasing fluid intake will lead to softer stools. This makes defecation easier. Fiber deficiencies may contribute to constipation. Fiber in the diet adds bulk to the stools which help them pass more readily through the intestines. 5. Incorrect: Ignoring the urge to defecate may increase the risk of constipation. Trying to defecate after a meal when peristalsis is increased may be helpful; however, if the urge occurs at other times, the client should go to the bathroom at that time to prevent constipation.

The nurse is working with a committee at the local school to develop an emergency preparedness plan for tornados. What should be included in the plan? 1. Identification of safe zones. 2. Methods for accounting for all people present in the building. 3. Warning system activation. 4. Identification of the gymnasium as the routine safe place. 5. Regular practice protocols.

1., 2., 3. & 5. Correct: Everyone should be aware of safe zones within the school. Personnel should be given this information and signs posted in safe zones. There must be systems in place to accurately determine the number of people in the building at any given time. There also must be a system in place to alert personnel and students of tornado warnings. Regular practice prepares everyone for an actual event.4. Incorrect: Gymnasiums are not considered safe places due to wide expanse of roof. Safe zones should be on interior walls, no windows, and a strong concrete floor if possible.

A client is preparing to be discharged after a total hip replacement. Which statements, if made by the client, would indicate to the nurse that teaching has been successful regarding prevention of hip prosthesis dislocation? 1. I should not cross my affected leg over my other leg. 2. I should not bend at the waist more than 90 degrees. 3. While lying in bed, I should not turn my affected leg inward. 4. It is necessary to keep my knees together at all times. 5. When I sleep, I should keep a pillow between my legs.

1., 2., 3. & 5. Correct: One of the most common problems after hip surgery is dislocation. Until the hip prosthesis stabilizes, it is necessary to follow these instructions for proper positioning to avoid dislocation. Flexion and movement of the leg on the affected side past midline should be avoided. 4. Incorrect: The knees should be kept apart at all times. This is called abduction and is needed to keep the new head of the femur (prosthetic device) in the acetabulum and therefore prevent hip dislocation until healing occurs and tissues are strong enough to hold the joint in place.

What should the nurse tell the parents of a newborn about a Guthrie test? 1. The purpose of this test is to determine the presence of phenylalanine in the blood. 2. A positive test indicates a metabolic disorder. 3. To conduct this test, a sample of blood is taken from the baby's heel. 4. An increase in protein intake can interfere with the test. 5. This test will be done when your baby is 6 weeks old.

1., 2., 3. Correct: These are true statements. A positive test indicates decreased metabolism of phenylalanine, leading to phenylketonuria. The normal level of phenylalanine in newborns is 0.5to 1 mg/dl. The Guthrie test detects levels greater than 4 mg/dl. Only fresh heel blood, not cord blood, can be used for the test. The main objective for diagnosing and treating this disorder is to prevent cognitive impairment. 4. Incorrect: A lack of protein intake can interfere with the test. The screening test is most reliable when the blood sample is obtained after the baby has ingested a source of protein. 5. Incorrect: Screening protocol involves testing the infant as close to discharge as possible but no later than 7 days after birth. If the infant is less than 24 hours old when the specimen is collected, a repeat test should be done before the infant is 2 weeks of age.

The charge nurse is making assignments for one RN and one LPN/VN on a pediatric unit. Which clients would be most appropriate for the charge nurse to assign to the RN? 1. 2 year old with asthma receiving IV medication. 2. 6 year old with new onset seizures. 3. 12 year old with colitis receiving TPN. 4. 2 month old with urinary tract infection. 5. 10 year old paraplegic needing assistance with bowel training.

1., 2., 3. Correct: These clients should be assigned to the RN as they will require more frequent assessment due to the nature of each diagnosis and have a potential for more rapid change in condition. Also, these clients may require skills by the RN that the LPN/VN could not do; for example, giving IV medications that asthma clients take; teaching the family about seizures, meds, and management; and administering TPN intravenously. 4. Incorrect: There is nothing in this option to indicate that the child is unstable. This assignment is appropriate for LPN/VN 5. Incorrect: This assignment is appropriate as the LPN/VN can provide care related to elimination needs.

What measures should the unit nurse initiate after admitting a client who had a chest tube inserted for pleural effusion of the right lung? 1. Place in semi-Fowler's position. 2. Connect to oxygen saturation monitor. 3. Assess respiratory status every 2 hours. 4. Prevent dependent loops in closed drainage unit tubing. 5. Maintain closed drainage unit at the level of the client's chest.

1., 2., 3., & 4. Correct: A pleural effusion is a collection of fluid in the pleural space that moves to the bottom of the chest cavity when upright. The semi-Fowler's position allows the client to be in an upright position to promote drainage and facilitate ease of respirations by promoting lung expansion. Since lung expansion is compromised with a pleural effusion, the oxygen level should be assessed using an oxygen saturation monitor. The client's respiratory status should be assessed at least every 2 hours: respiratory rate, work of breathing, breath sounds, pulse oximetry. The development of kinks, loops, or pressure on the drainage tubing can produce back pressure, which may force fluid back into the pleural space or interfere with the drainage.5. Incorrect: The closed drainage unit should be placed below the level of the client's chest to prevent drainage from flowing backward into the pleural space.

A school nurse is teaching a group of preteens with acne how to care for the skin. What points should the nurse include? 1. Wash face with soap and warm water. 2. Avoid using oily creams. 3. Do not use cosmetics that block sebaceous gland ducts. 4. Do not squeeze lesions. 5. Clean face vigorously with a terrycloth.

1., 2., 3., & 4. Correct: Washing the face frequently (at least twice a day) with mild soap or detergent and warm water will remove oil, dirt, and bacteria which increase inflammatory reactions and resulting acne. Oily creams and oil based cosmetics can block the ducts of the sebaceous gland ducts and the hair follicles making the acne worse. These should be avoided. Squeezing or picking at lesions will increase potential for infection and scarring. 5. Incorrect: Clean face gently, as trauma during acne breakouts may worsen the acne and cause scarring. When washing face, use hands, as terrycloth or other scrubbing material may cause acne sores to rupture.

The nurse is caring for a client admitted to the psychiatric unit with a diagnosis of major depression. What behaviors could the nurse expect upon assessment of this client? 1. Withdrawn behavior 2. Sitting in room, lights out, drapes closed 3. Unkempt appearance 4. Overeating 5. Severe insomnia Rationale

1., 2., 3., & 5. Correct: The client with severe depression has extremely low self-esteem and low energy levels and may just sit for hours. Depressed clients prefer to be alone and avoid social interactions. The room environment mimics the mood of the client (dark and gloomy). The client may not have the energy to bathe, change clothes, or even comb hair. The severely depressed person may have severe insomnia. However, sleeping too much is also a symptom of mild depression.4. Incorrect: The client who is severely depressed, as in the depressive disorder, usually has no appetite and loses weight. A mildly depressed client is more likely to overeat as a coping mechanism.

During an assessment interview with a client, what alternative healing modalities should the nurse inquire about? 1. "Tell me about your use of teas, herbs, and vitamins." 2. "What traditional or folk remedies are used in your family?" 3. "Do you meditate, pray, or use relaxation techniques for healing purposes?" 4. "What prescription medications are you taking?" 5. "What alternative therapies have you used?"

1., 2., 3., & 5. Correct: These are all inquiries the nurse should make when conducting an assessment interview in order to find out about alternative healing modalities. Alternative or complementary medicine is used to describe over 1800 therapies practiced around the world. Approximately 65 to 80% of the world's population use non-conventional (alternative) healing modalities. These alternative healing modalities can be such things as: Natural products (herbs, dietary supplements, etc.) mind and body practices (yoga, mediation, prayer, etc.), folk remedies and other non-traditional practices. 4. Incorrect: Prescription medications would be part of traditional, western medicine. Although the nurse needs to find out what prescription medications are being taken, it is not part of alternative medicine.

A nurse is planning to provide information regarding suicide to a high school assembly. What information should the nurse include? 1. Do not keep secrets for the suicidal person. 2. Express concern for a person expressing thoughts of suicide. 3. Teens often don't mean what they say, so only take suicide seriously if grades are dropping as well. 4. Inform group of suicide intervention sources. 5. Do not leave a suicidal person alone.

1., 2., 4. & 5. Correct: If a person reveals that suicide is being considered, this should never be kept secret. Help should be sought for the person immediately. It is also important to be direct and non-secretive with suicidal clients. It is appropriate to express concern for their thoughts. The use of empathy, warmth and concern indicates to the client that their feelings are being understood and viewed as real, which helps to build trust with the client. Resources for assistance are important to include in all health teaching programs. The teens need to know what resources are readily available if someone is considering suicide. The client contemplating suicide should not be left alone. This is for the client's safety until further assistance can be obtained 3. Incorrect: Most clients who commit suicide have told at least one person that they were contemplating suicide before thy actually committed the act. Therefore, suicidal comments should be considered important risk factors that require evaluation, and all comments should be taken seriously. Anyone expressing suicidal feelings needs immediate attention.

A case manager is assessing an unresponsive client diagnosed with terminal hepatic encephalopathy for equipment needs upon discharge home for hospice care. Which equipment should the case manager obtain for this client? 1. Alternating pressure mattress 2. Hospital bed 3. Walker 4. Suction equipment 5. Oxygen

1., 2., 4., & 5. Correct: An alternating pressure mattress will help to prevent pressure ulcers. The risk of respiratory compromise increases as the neurologic status deteriorates. A hospital bed is needed so that the head of the client's bed can be elevated to 30 degrees to ease respirations and decrease the work of breathing. The client with hepatic encephalopathy is unresponsive due to accumulation of toxins and may need suctioning if unable to clear secretions from the oropharynx. Hepatic encephalopathy frequently has associated bleeding varices. The increasing ascites leads to hypovolemia. Both of these conditions can result in hypoxemia for the client at the end stages of liver disease; therefore, oxygen therapy is provided. 3. Incorrect: As hepatic encephalopathy progresses and toxins accumulate, the client lapses into a coma. Therefore, the unresponsive client will not be ambulatory and would not need a walker.

A postoperative surgical client has a prescription for monitoring of intake and output (I&O). The I&O sheet has been picked up by the unlicensed assistive personnel (UAP) for the 7AM-3PM shift. Intake Output IV fluid-1025 mL Urine - 1350 mL PRBC-250 mL NG tube - 75 mL Jackson Pratt - 22 mL Calculate the client's output for the shift in mL.

1350 + 75 + 22 = 1447 mL

A client diagnosed with rheumatoid arthritis has been prescribed celecoxib. What should the nurse include in the client's education regarding this medication? 1. Do not take celecoxib with ibuprofen. 2. GI complaints and headache are among the most common side effects. 3. Drink a lot of water to offset the dehydration that may occur. 4. Notify the healthcare provider immediately if black stools are noted. 5. This medication provides relief of pain and swelling so you can perform normal daily activities.

1., 2., 4., & 5. Correct: Concomitant use of celecoxib with aspirin or other NSAIDs (for example, ibuprofen, naproxen, etc.) may increase the occurrence of stomach and intestinal ulcers. This would increase the risk of GI bleeders. GI complaints and headache are two of the most common side effects. The client should stop taking celecoxib and get medical help right away if the client notices bloody or black/tarry stools. This would be an indication of GI bleeding. This medication is a nonsteroidal anti-inflammatory drug (NSAID), which relieves pain and swelling. It is used to treat arthritis. The pain and swelling relief provided by this medication should help the client perform normal daily activities. 3. Incorrect: The client may develop fluid retention while taking this medication. They should decrease the intake of sodium to decrease fluid retention.

While examining a client's health history, which data indicates to the nurse that the client is at increased risk for developing cancer? 1. Family history 2. Alcohol consumption 3. Spicy diet 4. Human papillomavirus 5. Tobacco use

1., 2., 4., & 5. Correct: Family history of cancer increases the risk for having the same type of cancer. Alcohol and tobacco use increase the risk of cancer. When used together, they have a synergistic effect. Human papillomavirus (HPV) increases the risk of cervical, head, and neck cancers.3. Incorrect: Although there are some dietary factors associated with cancer development, a spicy diet does not necessarily increase the risk of cancer.

What should a nurse teach family members prior to them entering the room of a client who has agranulocytosis? 1. Meticulous hand washing is needed. 2. Do not visit if you have any infection. 3. The client must wear a mask. 4. Children under 12 may not visit. 5. Flowers are not allowed in the room.

1., 2., 4., & 5. Correct: Protective isolation is needed for this client because of the presence of a low white blood cell count. We are protecting the client from acquiring an infection. So any visitors will need to have meticulous hand washing prior to entering. The visitor should not enter if he or she has any type of infection. To decrease the risk of infection, small children should not visit. Even the mildest symptom of infection could be detrimental to the client. Flowers have bacteria and should not be brought into the room. 3. Incorrect: A mask must be worn by the visitor, not the client. The mask is worn by visitors to prevent a possible spread of an airborne infection to the immunocompromised client.

The nurse is caring for a client who has just arrived at the emergency department with suspected acute myocardial infarction. Which medications should the nurse administer immediately? 1. Oxygen 2. Heparin 3. Morphine 4. Sublingual nitroglycerin 5. Furosemide

1., 3., & 4. Correct: Initial management should take place immediately. According to the American Heart Association/Heart & Stroke Foundation of Canada and the American College of Cardiology, oxygen, SL nitroglycerin, morphine, and aspirin should be administered immediately. The initial goal of therapy for clients with an acute MI is to restore perfusion to the myocardium as soon as possible. Oxygen is appropriate and advisable when hypoxia is present. Pain from acute MI's may be intense and requires prompt administration of analgesia. Morphine sulfate is the medication of choice (2-4 mg every 5-15 minutes). Reducing the myocardial ischemia also helps reduce pain, so oxygen therapy and nitrates are main components of the therapy. The vasodilation effects of morphine and the nitroglycerin improve coronary blood flow and reduce myocardial ischemia. 2. Incorrect: Heparin is not part of the protocol within the guidelines and is not recommended at this time.5. Incorrect: Furosemide is not part of the protocol within the guidelines and is not indicated at this time.

The nurse is advising the family of a client receiving palliative care on alternative methods for pain control to be used in conjunction with pain medications. Which method should the nurse include? 1. Providing a back massage 2. Administering pain medication when pain is rated at 5 out of 10 3. Distracting with music 4. Exercise 5. Prayer

1., 3., & 5. Correct: These are types of alternative pain control that could be used in conjunction with traditional pain management. They can be used to provide relaxation and comfort; mind-body therapies such as meditation, guided imagery and hypnosis may be effective. Other measures may include: acupuncture, therapeutic touch, music therapy and spiritual practices such as prayer. These have been found to be effective in helping to reduce pain. 2. Incorrect: Pain medication is traditional, not alternative pain control. Also, pain medication should be provided prior to a rate of 5/10. 4. Incorrect: The client is likely not going to be able to exercise. Movement during pain may increase pain.

To reduce the risk of developing a complication following balloon angioplasty, the nurse should implement which measure? 1. Monitor cardiac rhythm 2. Assess the puncture site every 8 hours 3. Measure urinary output hourly 4. Prevent flexion of the affected leg 5. Avoid lifting buttocks off the bed

1., 3., 4., & 5. Correct: The primary healthcare provider should be notified of any rhythm changes or report chest pain/discomfort. These could be signs of re-occlusion. Decreased urinary output (UOP) could be due to poor renal perfusion, which can result from decreased cardiac output and shock. Frequent VS and UOP measurements are needed. Flexion should be avoided at the catheter access site to allow time for the clot to stabilize and reduce the risk of bleeding and hematoma formation. The client should avoid lifting the buttocks off the bed because this increases pressure at the insertion site which increases the risk of hematoma formation/bleeding.2. Incorrect: Assessments are needed more frequently than every 8 hours. Although policies may differ, assessment of the insertion site is usually every 15 minutes for 1 hour, every 30 minutes for 1 hour, and then hourly for 4 hours. More frequent monitoring may be required. During the assessment, the nurse should observe the catheter access site for bleeding or hematoma formation and should assess the peripheral pulses in the affected extremity.

The parents of a 4 year old child are concerned about whether the child will adapt to the newborn baby they are expecting in two weeks. What suggestions should the nurse make to assist with sibling adaptation? 1. Allow child to be one of the first to see the newborn. 2. Have child stay with parents during labor and delivery. 3. Arrange for one parent to spend time with the child while the other parent cares for the newborn. 4. Provide a gift from the newborn to give to the child. 5. Have child care for a doll.

1., 3., 4., & 5: These are good recommendations for the nurse to make to the parents in an effort to promote sibling adaptation. Make the 4 year old part of the process as much as possible. Demonstrate the importance of the child by allowing the child to see the baby first. Provide personal time with the 4 year old. This shows that the 4 year old is important to the family. The baby is providing a gift to the child which promotes a bond between the two and demonstrates to the child that he or she is important. Having a 4 year old care for a doll gets the child involved in caring for another. The child can learn what a newborn needs both physically and emotionally by imitating the parents. 2. Incorrect: This child is young and may not understand what is happening with their mother during contractions and delivery. Does not promote sibling adaptation. This is a 4 year old who would not understand what is going on during labor and delivery. It can be very frightening to the child and does nothing to support sibling adaptation.

The nurse is teaching a newly diagnosed diabetic about the action of regular insulin. The nurse verifies that teaching has been successful when the client verbalizes being at greatest risk for developing hypoglycemia at what time following the 8:00 a.m. dose of regular insulin? 1. 8:30 AM 2. 11:00 AM 3. 1:30 PM 4. 4:00 PM

2. Correct: 11:00 AM: Regular insulin peaks 2-3 hours after administration. Clients are at greatest risk for hypoglycemia when insulin is at its peak. 1. Incorrect: 8:30 AM: Rapid acting insulin will begin peaking in 30 minutes. 3. Incorrect: 1:30 PM: Intermediate acting insulin begins peaking at 4 hours. So at 1:30 PM this would be a time of worry. 4. Incorrect: 4:00 PM: At 4 PM you would still be worried about intermediate acting insulin. But you would also be worried about long acting insulin as well. Which starts to peak at 6 hours.

A nurse, assigned to take care of a client who is HIV positive, refuses the assignment, stating fear of personal injury. What action should the charge nurse take first? 1. Re-assign the client to a nurse who does not mind caring for HIV positive clients. 2. Inform the nurse that refusing client care is not acceptable nursing practice. 3. Have the nurse document rationale and support for refusing the client assignment. 4. Transfer the nurse to a unit where there are no HIV positive clients.

2. Correct. This action by the charge nurse demonstrates an understanding of the code of ethics for nurses. Any nurse who feels compelled to refuse to provide care for a particular type of client faces an ethical dilemma. The reasons given for refusal range from a conflict of personal values to fear of personal risk of injury. Such instances have increased since the advent of acquired immunodeficiency syndrome (AIDS) as a major health problem. The ethical obligation to care for all clients is clearly identified in the first statement of the Code of Ethics for Nurses. To avoid facing these moral and ethical situations, a nurse can follow certain strategies. For example, when applying for a job, one should ask questions regarding the client population. If one is uncomfortable with a particular situation, then not accepting the position would be an option. Denial of care, or providing substandard nursing care to some members of our society, is not acceptable nursing practice. As a professional, the nurse should provide the same level of care to every client, regardless of diagnosis, skin color, ethnicity or economic status. 1. Incorrect: This is not the best action for the charge nurse to take.The charge nurse should remind the nurse of the responsibility for the agency to provide nondiscriminatory care to all clients. The re-assignment of the client to another nurse does not resolve the ethical dilemma by the nurse refusing to provide care. 3. Incorrect: The nurse can inform the employer of the reluctance to care for a certain population, but must provide strong rationale and documentation to support the necessity for refusal of the assignment. Recognition by the organization of an individual nurse's right to refuse to care for a specific client population sets a major personnel precedent and will not be made lightly. A health care agency has a responsibility to provide care for all clients accepted into the organization. Due to this responsibility the nurse cannot be guaranteed that he/she will never be asked to provide care for the client in question. 4. Incorrect: This is generally a decision made on a level beyond the charge nurse. This is a last resort after documentation and consideration by management to accept the nurse's request. If honored, the nurse should expect to be transferred. However, the nurse may encounter the situation again.

The nurse in the emergency department suspects that a client's lesion is caused by anthrax. What assessment question is most important? 1. Have you traveled out of the United States recently? 2. Have you recently worked with any farm animals or any animal-skin products? 3. Have you experienced any gastrointestinal upset recently? 4. Have you eaten any home-canned foods recently?

2. Correct: Cutaneous anthrax may be contracted by working with contaminated animal-skin products. Anthrax is found in nature and commonly infects wild and domestic hoofed animals.1. Incorrect: Cutaneous anthrax is also found in the United States, so asking about travel abroad would not be necessary. 3. Incorrect: Cutaneous anthrax can be contracted by spores entering cuts or abrasions in the skin. This is cutaneous anthrax that causes edema, itching and macule or papule formation, resulting in ulceration. Ingestion of anthrax can cause GI symptoms such as nausea and vomiting, abdominal pain, and bloody diarrhea. Inhalation of anthrax may result in flu-like symptoms that progress to severe respiratory distress.4. Incorrect: This question would be appropriate if botulism were suspected in a client.

Which client will the charge nurse intervene on behalf when making rounds? 1. The client turned to left side 1 ½ hour ago. 2. Client who has been sitting in a chair for 2 ½ hours. 3. Client who is day one postop from hip replacement with abduction pillow in place. 4. The client who is in buck's traction with foot boots.

2. Correct: Limit sitting in a chair to less than 2 hours. Prolonged sitting or lying in one position predisposes the client to skin breakdown and other hazards of immobility. 1. Incorrect: Clients should be turned at least every 2 hours, so this client is within the acceptable time frame for being turned. 3. Incorrect: The client with hip replacement needs the abduction pillow. This prevents dislocation of the hip prosthesis by helping to maintain the femoral head component in correct position. 4. Incorrect: The client in buck's traction needs foot boots to avoid foot drop. Therefore, this is an appropriate intervention.

The nurse is teaching the Type II diabetic about monitoring average blood glucose levels over time. The nurse evaluates teaching has been effective when the client verbalizes the need to return to the clinic for which test? 1. Glucose tolerance test 2. Glycosylated hemoglobin 3. Glucose-6-phosphate dehydrogenase 4. Fasting blood glucose

2. Correct: Glycosylated hemoglobin (also known as hemoglobin A1C) tests the average blood glucose over 90 days, or 2-3 month time period. Specifically, this test measures the percentage of hemoglobin that is coated with blood sugar (glycated). 1. Incorrect: Glucose tolerance test will show the immediate tolerance or response, to a glucose load. This test is often used to screen for gestational diabetes, and can be used to screen for type II diabetes. However it does not indicate what the blood glucose levels have been over time. 3. Incorrect: Glucose-6-phosphate is an enzyme that assists in maintaining the level of glutathione in erythrocytes to help protect against oxidative damage and breakdown of red blood cells (hemolytic anemia). Deficiency in glucose-6-phosphate dehydrogenase is linked to a genetic defect. This test does not measure blood glucose levels. 4. Incorrect: Fasting blood glucose tests immediate glucose levels, after an overnight fast. This does not indicate what the blood glucose levels have been over time.

The nurse is caring for a client diagnosed with herpes varicella zoster. What pharmacologic agent should the nurse anticipate the primary healthcare provider will prescribe? 1. Metronidazole 2. Acyclovir 3. Ceftriaxone 4. Ampicillin

2. Correct: Herpes varicella zoster is a virus that causes chickenpox in children and shingles in adults. An antiviral such as acyclovir, is indicated.1. Incorrect: Herpes varicella zoster is a virus; an antiviral is indicated, not a nitromodazole antimicrobial, such as metronidazole. Metronidazole may have additional classifications such as: amebecide, antibiotic, antibacterial, etc.3. Incorrect: Herpes varicella zoster is a virus; an antiviral is indicated, not an antibiotic such as ceftriaxone.4. Incorrect: Herpes varicella zoster is a virus; an antiviral is indicated, not an antibiotic such as ampicillin.

A client arrives in the emergency department after severely lacerating the left hand with a knife. HR 96, BP 150/88, R 36. The client is extremely anxious and crying uncontrollably. Based on this assessment, the nurse should anticipate that this client is likely in which acid base imbalance? 1. Respiratory acidosis 2. Respiratory alkalosis 3. Metabolic acidosis 4. Metabolic alkalosis

2. Correct: Hyperventilation due to anxiety, pain, shock, severe infection, fever, liver failure can lead to respiratory alkalosis. With each of these, the client loses too much CO2. The reduction of CO2 creates an excessive loss of acid, resulting in an alkalotic state. Since the problem is respiratory, it is respiratory alkalosis. 1. Incorrect: This problem is respiratory, but there is excessive CO2 loss. CO2 combines with water to form an acid. If too much of the CO2 is lost, the result of the acid forming substance loss would be alkalosis-Not acidosis. 3. Incorrect: The problem in this situation is respiratory in origin and has acid loss. Therefore, it is not metabolic nor acidotic in nature. 4. Incorrect: The problem in this situation is the excessive loss of CO​2 from the respiratory system secondary to hyperventilation. Although the CO2 loss creates an alkalotic state, it is respiratory, not metabolic in origin.

A client, hospitalized with possible acute pancreatitis secondary to chronic cholecystitis, has severe abdominal pain and nausea. The client is kept NPO, an NG tube is inserted, and IV fluids are being administered. What is the rationale for the client being NPO with an NG tube to low suction? 1. Relieve nausea 2. Reduce pancreatic secretions 3. Control fluid and electrolyte imbalance 4. Remove the precipitating irritants

2. Correct: In clients with pancreatitis, the pancreatic enzymes cannot exit the pancreas. These enzymes, when activated, begin to digest the pancreas itself. The enzymes become activated in the pancreas when fluid or food accumulates in the stomach. The goal in treating this client is to stop the activation of the pancreatic enzymes. Treatment is focused on keeping the stomach empty and dry. This allows the pancreas time to rest and heal. Note: Autodigestion (pancreas digesting itself) is painful for the client and can lead to other problems such as bleeding. 1. Incorrect: The primary purpose of the NG tube to suction is to keep the stomach empty and dry to decrease pancreatic enzyme production, not to relieve nausea. 3. Incorrect: Because gastric contents are removed, the NG tube to suction may lead to fluid and electrolyte disturbances rather than helping to control them. 4. Incorrect: Although the food in the stomach causes the pancreatic enzymes to become activated in the pancreas due to the obstruction, the food is not considered an irritant. Precipitating irritants are not a part of the pathophysiology occurring with pancreatitis.

During the insertion of a urinary catheter, the tip of the catheter touches the client's thigh. What action should the nurse take? 1. Wipe the tip of the catheter with alcohol. 2. Call for another urinary catheter and a pair of sterile gloves. 3. Insert the catheter and obtain a prescription for antibiotics. 4. Leave the room to obtain another sterile urinary catheter kit.

2. Correct: Indwelling catheter insertion is a sterile procedure. If contamination occurs, do not turn back on sterile field. Get on the call light to request another urinary catheter and sterile gloves to continue the procedure. Continuing the procedure with contaminated equipment would jeopardize the client's safety. 1. Incorrect: This is a sterile procedure. The catheter needs to be replaced because it is no longer sterile. The client would be at high risk of developing a urinary tract infection. The catheter cannot be made sterile by the use of alcohol. 3. Incorrect: This is a sterile procedure. The catheter needs to be replaced because it is no longer sterile. Inserting the now non-sterile catheter puts the client at risk for infection. There is no reason at this time to start antibiotics. 4. Incorrect: The catheter is contaminated, but the sterile field is still okay. It is more cost efficient to have someone bring the nurse another catheter and pair of sterile gloves rather than getting an entire sterile kit.

A new nurse is preparing to give a medication to a nine month old client. After checking a drug reference book, the nurse crushes the tablet and mixes it into 3 ounces of applesauce. The new nurse proceeds to the client's room. What priority action should the supervising nurse take? 1. Tell the new nurse to recheck the drug reference book before administering the medication. 2. Suggest that the new nurse reconsider the client's developmental needs. 3. Check the prescription order and the client dose. 4. Observe the new nurse administer the medication.

2. Correct: Mixing medication with applesauce is appropriate in some circumstances, but the volume of 3 ounces is excessive for a nine month old. The nurse will want to make sure the client gets all of the medication. Additionally, applesauce may or may not have been introduced into the diet, and it is inappropriate to introduce a new food during an illness. 1. Incorrect: There is nothing in the stem about a problem with the medication dose or route. 3. Incorrect: There is nothing in the stem about a problem with the medication dose or route. 4. Incorrect: This is an appropriate action. However, it is not the priority over ensuring that the new nurse knows how to appropriately prepare the medication for this client.

A client diagnosed with major depression is admitted to the psychiatric unit for electroconvulsive therapy (ECT). The client asks the nurse, "How many of these treatments do you think I will need?" What is the nurse's best response? 1. That is a question you need to discuss with your primary healthcare provider. 2. Everyone responds differently, but on average clients need 6-12 treatments. 3. You will need to take a treatment every month for at least a year. 4. Let's just take one treatment at a time, shall we?

2. Correct: Most clients require an average of 6 to 12 treatments, but some may require up to 20 treatments. These treatments are generally given two to three times per week for three to four weeks. The number of treatments required depends on the severity of the symptoms and how quickly the client improves. 1. Incorrect: The nurse should be able to answer this question based upon the generally accepted regimen for electroconvulsive therapy (ECT). 3. Incorrect: Treatments are usually administered every other day (three times per week). Since the average number of treatments is 6-12, it only takes a couple of weeks to a month, on average for the regimen. Treatments are performed on an inpatient basis for those who require close observation and care, but can be done on an outpatient basis for some clients. 4. Incorrect: This is poor therapeutic communication. The nurse did not answer the question and is belittling. The client has a right to be able to make informed decisions regarding care being provided.

A client diagnosed with major depression has been taking a selective serotonin reuptake inhibitor for the past 6 weeks. When visiting the mental health center, the nurse discusses the medication and response with the client. The nurse's assessment reveals that the client is confused about the date and about the prescribed dosage of the medication. Which question would be most important for the nurse to ask to further assess the situation? 1. Are you having trouble sleeping at night? 2. Do you have periods of muscle jerking? 3. Are you having any sexual dysfunction? 4. Is your mood improving?

2. Correct: Myoclonus, high body temperature, shaking, chills, and mental confusion are some of the symptoms of serotonin syndrome. This client may be having symptoms of this adverse reaction which, if severe, can be fatal. 1. Incorrect: Sleep disturbances are common with depression. Selective serotonin reuptake inhibitors (SSRIs) may cause insomnia; however, there is a more pertinent question needed for assessment of this client. You should be concerned with the more serious or life-threatening issue. 3. Incorrect: Sexual dysfunction may occur with the SSRIs; however, the client is exhibiting significant symptoms of an adverse reaction which would take priority. 4. Incorrect: The response to the SSRI medications is important; however, there is a more significant issue in this case. The possible serotonin syndrome is a serious situation that would be the priority for the nurse to address.

The nurse is giving discharge instructions to an Asian client following a colonoscopy. During the instructions, the client stares directly at the floor, despite being able to speak English. Based on the client's body language, how would the nurse classify this behavior? 1. Embarrassment. 2. Attentiveness. 3. Disinterest. 4. Confusion.

2. Correct: Nurses must be aware of clients' specific cultural or religious beliefs in order to provide appropriate care and discharge planning. Asian societies have a deep respect for others and making eye contact with the nurse would be considered rude and offensive. The nurse is considered superior to the client, so direct eye contact with a superior shows a lack of respect. This client is displaying attentiveness while also showing respect for the nurse. 1. Incorrect: There is nothing in the question to suggest the client is embarrassed. In Asian cultures, making eye contact is considered disrespectful to the superior; therefore, this client's demeanor is a respectful display of cultural influences. 3. Incorrect: The client's body language does not suggest disinterest. Although staring downward, this client does not display other signs of disinterest. A culturally aware nurse understands that the client's Asian background impacts this behavior and conveys the meaning of respect for the nurses' position. 4. Incorrect: The question indicates that the client does speak English. There is no indication that the client is confused or does not comprehend the discharge instructions. There is a more specific cultural basis for the client's behavior.

A child is being admitted with possible rheumatic fever. What assessment data would be most important for the nurse to obtain from the parent? 1. 102° F (38.89° C) temperature that started 2 days previously. 2. History of pharyngitis approximately 4 weeks ago. 3. Vomiting for 3 days. 4. A cough that started about 1 week earlier.

2. Correct: Rheumatic fever is often the result of untreated or improperly treated group A β-hemolytic streptococcal infections (GABHS), such as pharyngitis. Therefore, the history of pharyngitis or upper respiratory infection is a key assessment finding for establishing a diagnosis of rheumatic fever. Subsequent development of rheumatic fever usually occurs 2 to 6 weeks following the GABHS, so the assessment should include a remote history of pharyngitis. 1. Incorrect: The fever with rheumatic fever is usually low grade and is considered a minor manifestation of rheumatic fever. 3. Incorrect: Vomiting is not a commonly associated symptom with rheumatic fever and is not considered a major manifestation of rheumatic fever. Although the child may have a history of vomiting, this finding would not be specific to rheumatic fever. 4. Incorrect: A cough is not an associated symptom of rheumatic fever. The time frame for the development of rheumatic fever is not appropriate if the cough started 1 week earlier, even if it had been associated with an upper respiratory streptococcal infection.

A newly admitted client with schizophrenia has an unkempt appearance and needs to attend to personal hygiene. Which statement by the nurse is most therapeutic? 1. A shower will make you feel better. 2. It is time to take a shower. 3. Have you thought about taking a shower? 4. I need you to take a shower.

2. Correct: Schizophrenia is a thought disorder. Many clients with schizophrenia are concrete thinkers and have difficulty making decisions. The nurse needs to be direct, clear and concise in communicating with the client. This is a direct, clear and concise statement that guides the client to perform the needed activity. 1. Incorrect: Many clients with schizophrenia are concrete thinkers. The nurse needs to be direct, clear and concise in communicating with the client. The client may not comprehend how the shower improves the overall sense of well-being and would remain reluctant to take the shower. 3. Incorrect: Clients diagnosed with schizophrenia often have trouble making decisions. The client needs to be guided with simple, direct instructions. 4. Incorrect: This focuses on the nurse's need, not the client's need. Do not select answers that focus on the nurse. This does not improve the client's decision making ability nor does it provide guidance to the client for meeting the hygiene needs.

The nurse is caring for a client admitted to the skilled nursing unit approximately 3 months ago. Since admission, the client has lost 8 pounds. There have been no documented changes in the client's physical health. Which strategy may help to improve caloric intake for this client? 1. Encourage the client to eat meals in the room. 2. Take the client to the dining room for all meals. 3. Provide a high protein supplement 30 minutes before meals. 4. Ask the unlicensed assistive personnel to feed the client at each meal.

2. Correct: The client may be lonely and miss the interaction with others, but reluctant to go to the dining room. Eating with others may help to improve appetite and intake of food. The nurse can actively seek out the client and take this client to the dining room. Simply encouraging the client to go to the dining room may not be sufficient to get the client to go. 1. Incorrect: Eating alone may actually lead to reduce food intake. Eating is also a social activity. 3. Incorrect: A high protein supplement may increase caloric intake; however, to give that to the client 30 minutes before a meal will interfere with food intake at mealtime. 4. Incorrect: Assisting the client is important if the client cannot do it, however, there is no data to suggest that the client cannot eat independently. It is important to help the clients maintain their maximum level of independence.

The nurse is caring for a client who is receiving a prostaglandin agonist for the treatment of glaucoma. Which comment by the client indicates a lack of understanding of the treatment regimen? 1. I must only use the drops in the eye with the increased pressure. 2. My eyes may be different colors, so I will use the drops in both eyes. 3. I must be careful not to overmedicate even if it is just an eye drop. 4. The eyelashes in the eye with the higher pressure may get longer.

2. Correct: The color of the iris may darken in the eye being treated; however, it is important that the client understand that drops should not be placed in the unaffected eye. Prostaglandins cause increased permeability in the sclera to aqueous fluid. So, as the prostaglandin agonist increases this activity, the outflow of aqueous fluid increases and the ocular pressure decreases. Administering the drops in the unaffected eye may result in a subnormal intraocular pressure.1. Incorrect: This comment shows adequate understanding. The client should only treat the eye with the increased pressure.3. Incorrect: This comment demonstrates that the client does understand the treatment regimen. Overmedicating the affected eye could reduce the intraocular pressure too much. 4. Incorrect: This comment shows understanding. The lashes in the eye being treated will lengthen as opposed to the untreated eye. The changes of the eyelashes (increased length, thickness, pigmentation and number of lashes) are typical with these eye drops and are viewed as a benefit by many clients.

The client at the mental health center has voiced suicidal thoughts and has access to firearms at home. Which action by the nurse is priority? 1. Empathize with the client and listen to feelings. 2. Inform the family and ask them to remove the guns. 3. Chart the thinking pattern and make a follow up appointment. 4. Ask the client to return to the clinic tomorrow for further evaluation.

2. Correct: The family should be notified. Suicidal thinking is one condition that necessitates breach of confidentiality. The client has identified a plan and has access to firearms; therefore, the family should remove them from the house. Client safety is a priority. This client will likely be directly admitted to the hospital.1. Incorrect: This is appropriate; however, client safety is priority at this time. Suicide risk is higher when a plan is expressed and lethal means are available. 3. Incorrect: Charting the thinking pattern is an appropriate action; however, the priority at this time is the client's safety. Suicide risk is higher when a plan is expressed and lethal means are available. Making a follow up appointment would be delaying care for the immediate action that is needed to protect the client.4. Incorrect: Suicide risk is higher when a plan is made and lethal means are available. Asking the client to return to the clinic tomorrow would be delaying care for the immediate action that is needed to protect the client. It is likely that this client will be admitted directly to the hospital.

A client has been taught guided imagery as a method to relieve pain. How should the nurse first assess for pain relief after completion of guided imagery by the client? 1. Assess vital signs 2. Use of pain intensity scale 3. Ask client to describe the pain 4. Observe ability to perform activities of daily living

2. Correct: The use of pain intensity scales is an easy and reliable method of determining the client's pain intensity. 1. Incorrect: Although respiratory and heart rate may decrease with guided imagery and pain reduction, the most objective measure is to ask the client to rate the pain. 3. Incorrect: First, ask the client if pain is present. If present, the client should be asked to rate the pain. Once pain has been rated, the client should be asked to describe the pain. 4. Incorrect: The client may be able to perform activities of daily living and still have pain. Therefore, this would not be an accurate means of assessing pain relief.

Which assessment finding would indicate to a nurse that a client receiving chemotherapy may have difficulty maintaining proper nutrition? 1. Fatigue 2. Mucositis 3. Neutropenia 4. Diarrhea

2. Correct: Ulcerations in the oral cavity can make it difficult to chew food or be intolerant to certain foods due to discomfort and pain. Intake may be inadequate as a result of this. 1. Incorrect: Fatigue may make the client tire easily, but ulcerations in the oral cavity will be the primary reason for not wanting to eat. 3. Incorrect: Neutropenia leads to infection. This does not alter intake. 4. Incorrect: Diarrhea may need to be treated by making diet changes. However, the maintenance of nutrition should be focused on intake. The impact of the mucositis should be considered first for maintaining proper nutrition.

The nurse is caring for a client in the emergency department after a violent altercation with her husband. She describes increasingly violent episodes over the past 10 years. She says, "This is the last time he will hit me." Which response by the nurse demonstrates understanding of the violence cycle? 1. When you leave, you don't have to worry anymore. 2. You are at greatest risk when you leave. 3. That is the best decision you can make. 4. I am glad that you won't be hurt ever again.

2. Correct: Violence is likely to escalate and may become lethal when the spouse leaves the abusive partner. The risk of death or injury is highest at the time the abused person decides to leave the abusive relationship or shortly after leaving. 1. Incorrect: Just because the victim leaves does not guarantee that the abuser will not follow or find her. The threat of injury or death increases at the time the abused person leaves. This response is giving false reassurance to the abused person. 3. Incorrect: The client should be praised; however, there are risks with both leaving and staying. The client should be informed. The nurse should acknowledge the fear of staying in the relationship and guide the client to resources that can be used to help make informed decisions. 4. Incorrect: Leaving the home and the perpetrator do not guarantee cessation of violence. Again, this only provides false reassurance that the abuser will not find the client and inflict harm.

Which statement, made by a client scheduled for a total laryngectomy, indicates to the nurse a need for further preoperative teaching? 1. After the surgery, I will breathe only through a hole in my neck. 2. My wife will have to get a hearing aid because I will not be able to talk above a whisper. 3. I must have smoke detectors installed at home since I may not be able to smell after surgery. 4. After surgery, I will have a tube going through my nose to my stomach for feeding.

2. Correct: With a total laryngectomy, the vocal cords are removed. The entrance to the trachea is closed, so no air moves upward into the throat or mouth areas. The client will not be able to speak or whisper. The client's wife does not need a hearing aid, so further teaching is necessary. 1. Incorrect: The client will breathe through a hole in his neck (tracheostomy) for the rest of their life. This is a true statement by the client. We are looking for the false statement. 3. Incorrect: Since the entrance to the trachea is closed, the client can no longer move air through the nasopharynx. Therefore, the capacity to smell may be diminished or lost. The ability to smell remains intact because the sensory nerves in the nose are not impacted by the surgery. However, in order to smell normally, air must pass over the sensory cells which is not occurring in this case. This is a true statement by the client. We are looking for the false statement.4. Incorrect: During surgery, a feeding tube is placed in the stomach or jejunum to assist in nutritional requirements until the surgical area in the throat is healed. With a total laryngectomy, the client will eventually be able to eat because the trachea and esophagus are completely separate from each other. This is a true statement by the client. We are looking for the false statement.

A group of women ask a community health nurse how to prevent stress incontinence. What points should the nurse teach these women? 1. Limit alkaline foods. 2. Avoid caffeine. 3. Maintain a healthy weight. 4. Eat less fiber. 5. Perform high-impact exercise.

2., & 3. Correct: Fluids containing caffeine, carbonation, alcohol or artificial sweeteners act as irritants to the bladder wall and should be avoided. Acidic foods, such as citrus fruits, are also irritants. Obesity can cause increased pressure on the bladder, leading to incontinence. 1. Incorrect: Acidic foods, not alkaline, are bladder irritants and should be avoided. 4. Incorrect: The client should eat more fiber (not less) to prevent constipation, which can put pressure on the bladder and be a cause of urinary incontinence. 5. Incorrect: High-impact exercise puts pressure on the pelvic floor muscles and can increase leakage. Try Pilates, a gentle method of stretching and strengthening core muscles, which has become a more popular treatment for stress incontinence.

Which statements should a nurse make when educating a client about advance directives? 1. Used as guidelines for client treatment should the client's family deem them necessary. 2. Legally binding document. 3. Should be documented in the client's medical record as to whether or not the client has an advance directive. 4. Specifies a client's wishes for healthcare treatment should the client become incapacitated. 5. Allows the client's spouse to make end-of-life decisions.

2., 3. & 4. Correct: Advance directives are legally binding documents. Documentation is required in the medical record as to whether an advance directive exists. If one exists, a copy should be placed in the medical record. The document is prepared by the client detailing wishes for treatment should the client become unable to make informed healthcare decisions. 1. Incorrect: The family's wishes for treatment of the client do not take the place of or negate the client's advance directive.5. Incorrect: The spouse's wishes for treatment of the client do not take the place of or negate the client's advance directive.

Which factors should the nurse include when teaching a parent about risk factors for otitis media? 1. Breast-feeding 2. Contact with siblings 3. Day care attendance 4. Season of the year 5. Age over 5

2., 3. & 4. Correct: Contact with siblings, day care attendance, and season of the year all increase a child's risk of developing otitis media. Otitis media usually follows or accompanies an upper respiratory infection or the common cold. The exposure to upper respiratory infections is increased when other siblings are in the home and when the child attends daycare. More upper respiratory infections occur during times when the climate changes and during the winter months. 1. Incorrect: Breast-feeding decreases the incidence of otitis media. Ear infections are more common in children who drink from bottles or sippy cups, especially when lying on their back. 5. Incorrect: Age under 5 is a risk factor. The Eustachian tube is shorter, narrower, and more vulnerable to blockage in the younger children. It also lies more horizontal and does not drain as well as older children and adults. This, along with immature immune systems, puts the younger child at higher risks for otitis media.

A nurse educator is explaining the Health Insurance Portability and Accountability Act (HIPAA) of 1996 to a group of nursing students. What points about HIPAA should the nurse educator include? 1. Primary healthcare providers employed at the facility where a client receives treatment can legally access any client's health information at any time. 2. Health related information revealed by a client to healthcare personnel must be kept confidential. 3. The client has the right to access personal healthcare records and to obtain copies of those records. 4. A client's information can be revealed only with the client's permission, or when the primary healthcare provider or facility is required by law to do so. 5. Unlicensed assistive personnel employed where a client receives treatment can legally access any client's health information at any time.

2., 3. & 4. Correct: HIPAA is federal legislation enacted to protect client health information and privacy. Any information the client reveals to healthcare personnel must be kept confidential. Clients have the right to access their personal healthcare records and to obtain copies of the records. A client's health information can be revealed only with the client's permission, or when a healthcare provider or facility is required to do so by law. 1. Incorrect: Healthcare personnel do not have the right to access a client's medical records or health information without treatment necessity.5. Incorrect: Unlicensed assistive personnel do not have the right to access a client's medical record or health information.

The nurse wants to provide anticipatory guidance for a group of young parents who have children between the ages of 18 months to 3 years. What points about the next year should the nurse be sure to provide these parents? 1. Be strict and rigid with toilet training, rather than being accepting and letting the child lead the training. 2. Tell the parents about the importance of letting the child do tasks alone. 3. Provide finger foods for the child to eat. 4. Your child will want you to provide emotional support when needed. 5. Assist your child with all tasks to promote independence.

2., 3. & 4. Correct: Letting the child do things on their own will promote a sense of self control and independence during this stage of autonomy versus shame and doubt. Finger foods allow for independence with eating and builds a sense of autonomy. At this age, the child becomes increasingly aware of separateness from the parent. The need is for the parent to be available for emotional support when needed. However, if emotional needs are inconsistently met or if the parent rewards clinging, dependent behaviors and withholds nurturing when the child demonstrates independence, feelings of rage and fear of abandonment may develop in adulthood. The support provided by the parent can lessen feelings of anxiety for the child when the emotional presence is needed. 1. Incorrect: Strict toilet training can result in retention of feces and constipation. In addition, strict toilet training practices before the child is ready can result in frustration and shame. 5. Incorrect: Assisting with all tasks will promote dependence. This does not give the child opportunities to perform age-appropriate tasks independently and gain a sense of autonomy. Notice the word "all"? This conveys a thought or concept that has no exceptions. Words such as just, always, never, all, every, none, and only are absolute and place limits on the statement that generally is considered correct. Statements including these words generally make the statement false as the statement is general and broad and does not allow for exceptions.

Which signs and symptoms would the nurse expect to see in a client who has taken prednisone for two months? 1. Weight loss 2. Decreased wound healing 3. Hypertension 4. Decreased facial hair 5. Moon face

2., 3. & 5. Correct: Decreased wound healing is a side effect with prolonged steroid use due to the immunosuppressive effects. All steroid medications, such as prednisone, can lead to sodium retention which then leads to dose related fluid retention. Hypertension is seen due to this fluid and sodium retention. Cushingoid appearance (moon face) is a side effect that is created from the abnormal redistribution of fat from prolonged steroid use. 1. Incorrect: Within one month after corticosteroid administration, weight gain is seen rather than weight loss.4. Incorrect: Facial and body hair increase with prolonged steroid use. This excessive growth of body hair, known as hirsutism, is one of the numerous potential side effects of prednisone.

A client's last two central venous pressure (CVP) readings were 13 cm of water. The nurse would expect the client to manifest which associated signs and symptoms? 1. Dry oral mucus membranes 2. Tachypnea 3. Orthostatic hypotension 4. Rales in the posterior chest 5. Jugular vein distention 6. Weight gain

2., 4., 5. & 6. Correct: The normal range for CVP is 2-8 cmH​2O or 2-6 mmHg. Therefore, the readings of 13 cmH​2O are high and may be the result of fluid volume excess. The signs and symptoms of FVE include: tachynea, rales, and jugular vein distention from the increased volume and preload. Acute weight gain is one of the best indicators of FVE due to circulatory overload. 1. Incorrect: The CVP is high and correlates with fluid volume excess. Dry oral mucous membranes indicate fluid volume deficit. 3. Incorrect: The CVP is high and correlates with fluid volume excess. Orthostatic hypertension indicates fluid volume deficit.

A nurse is caring for a client who delivered a baby vaginally two hours ago. What signs and symptoms of postpartum hemorrhage should the nurse report to the primary healthcare provider? 1. Two blood clots the size of a dime. 2. Perineal pad saturation in 10 minutes. 3. Constant trickling of bright red blood from vagina. 4. Oliguria 5. Firm fundus

2., 3., & 4. Correct: Lochia should not exceed an amount that is needed to partially saturate four to eight peripads daily, which is considered a moderate amount. Perineal pad saturation in 15 minutes or less is considered excessive and is reason for immediate concern. Saturation of a peripad in one hour is considered heavy. Also, trickling of bright red blood from the vagina can indicate hemorrhage and is often a result of cervical or vaginal lacerations. Bright red blood indicates active bleeding. Oliguria is a sign of fluid volume deficit. As blood volume goes down, renal perfusion decreases and urinary output (UOP) decreases. The kidneys are also attempting to hold on to what little fluid volume is left. 1. Incorrect: A few small clots would be considered normal and occur due to pooling of the blood in the vagina. Passage of numerous or large blood clots (larger than a quarter) would indicate a problem. 5. Incorrect: We worry about a boggy uterus. Uterine atony is a major cause of postpartal hemorrhage. The fundus feels firm as the uterus and uterine muscles contract to reduce the blood loss.

The school nurse has identified a large outbreak of viral conjunctivitis among one middle school class and plans to educate these students on this illness. Which data should the nurse be sure to include? 1. Use personal handkerchief to wipe the eye of discharge. 2. Light cold compresses over the eyes several times a day will ease discomfort. 3. Do not share towels or linens. 4. Discard all makeup and use new makeup after infection resolves. 5. Wash hands frequently with soap and water.2., 3., 4. & 5. Correct: All of these measures will promote comfort and decrease risk of transmitting infection. Clients should also avoid touching the eyes and shaking hands/touching other. Cool compresses provide symptomatic relief. 1. Incorrect: Use a new tissue every time you wipe the discharge from the eye. You can dampen the tissue with clean water to clean the outside of the eye. If a personal handkerchief is used, reinfection can occur.

2., 3., 4. & 5. Correct: All of these measures will promote comfort and decrease risk of transmitting infection. Clients should also avoid touching the eyes and shaking hands/touching other. Cool compresses provide symptomatic relief. 1. Incorrect: Use a new tissue every time you wipe the discharge from the eye. You can dampen the tissue with clean water to clean the outside of the eye. If a personal handkerchief is used, reinfection can occur.

The nurse on a neuro rehabilitation unit is caring for a client with a T4 lesion. The client suddenly reports a severe, pounding headache. Profuse diaphoresis is noted on the forehead. The blood pressure is 180/112 and the heart rate is 56. What interventions should the nurse initiate? 1. Place client supine with legs elevated. 2. Assess bladder and bowel for distention. 3. Examine skin for pressure areas. 4. Eliminate drafts. 5. Remove triggering stimulus. 6. Administer hydralazine if BP does not return to normal.

2., 3., 4., 5. & 6. Correct: The client is experiencing autonomic dysreflexia, which is a potentially dangerous syndrome that can develop in clients with spinal cord injuries. The cause of autonomic dysreflexia with these associated symptoms is a strong sensory or noxious stimulus. The most common stimulus is bowel, bladder distention, or irritation. Any painful, irritating or strong stimulus including environmental temperature changes, drafts, etc. can trigger autonomic dysreflexia. It is considered a medical emergency and must be promptly treated. 1. Incorrect: The client should be placed immediately in a sitting position to lower blood pressure. The supine position with the legs elevated could increase the BP to higher and more dangerous levels.

What interventions should the nurse initiate to keep the airway free of secretions in a client with pneumonia? 1. Evaluate results of ABG's and report abnormal findings. 2. Increase oral intake to at least 2000 mL/day. 3. Administer a cough suppressant medication. 4. Educate client on incentive spirometry. 5. Perform percussion to affected area.

2., 4., & 5. Correct: Liquefy secretions by increasing oral intake to at least eight, 8 ounce glasses of liquid/day unless fluid restrictions are required. Incentive spirometry helps keep alveoli open and prevents further pneumonia and atelectasis. Prescribed percussion can assist with loosening secretions for expectoration.1. Incorrect: This does not get rid of secretions. This monitors respiratory effectiveness.3. Incorrect: The nurse knows that client needs to expectorate the sputum to remove bacteria or prevent bacterial growth. If the cough is suppressed, the sputum will remain in the lungs, providing a medium for bacterial growth.

The nurse is working in a long term care facility. What actions by the nurse are appropriate when taking a telephone prescription from a primary healthcare provider? 1. Document the prescription prior to the end of the shift. 2. Explain to the pimary healthcare provider that nurses cannot take telephone prescriptions. 3. Repeat the prescription back to the primary healthcare provider prior to hanging up. 4. Transcribe the prescription in the client's record. 5. Ask the primary healthcare provider to wait and write the prescription during rounds.

3. & 4. Correct: Whenever a verbal or telephone prescription is given, the nurse is to transcribe the prescription, and then read it back to the prescribing primary healthcare provider at the time the prescription is given for validation of accuracy of the prescription received. Otherwise an error may occur.1. Incorrect: Errors are more likely to be made if documentation is not made at the time the prescription is received.2. Incorrect: Nurses can take telephone prescriptions; however, safety measures include writing down the prescriptions immediately and repeating the prescriptions to the primary healthcare provider.5. Incorrect: Asking the primary healthcare provider to wait until rounds is not appropriate, as nurses can take telephone prescriptions with appropriate safety measures to ensure accuracy.

The nurse is caring for a client who was admitted to the hospital following a severe motor vehicle crash (MVC) in which the client was trapped in the car for several hours. The client is being closely monitored for the development of renal failure. Which assessment finding would warrant immediate reporting? 1. Creatinine 1.1 mg/dl (97.24 mmol/L) 2. Urinary output of 150 mL per hour. 3. Gradual increase of BUN levels. 4. Calcium levels of 9.0 mg/dL (2.25 mmol/L)

3. Correct. Gradual accumulation of nitrogenous wastes results in elevated BUN and serum creatinine. This is an indication of impaired renal function. 1. Incorrect. This is a normal creatinine level. Gradual accumulation of nitrogenous wastes from impaired renal function results in elevated BUN and serum creatinine.2. Incorrect. This is a normal output level. This level alone would not necessarily be an indicator of acute renal failure and that value alone would not warrant reporting it to the primary healthcare provider.4. Incorrect. Calcium level of 9.0 mg/dL (2.25 mmol/L) is considered normal. When observing for renal functioning you would assess the BUN and creatinine levels. In addition, the calcium level may drop (hypocalcemia) in renal failure inverse relationship change due to the rising serum phosphate levels. However, the calcium level presented is within normal limits (WNL).

A client is hospitalized for recurrent angina with hypertension and has been started on new medications. When reviewing the admission forms, the nurse should immediately question which prescription? Primary Healthcare Provider Prescriptions Spironolactone 50 mg. P.O. once daily. Metoprolol 25 mg. P.O. once daily. Diltiazem 120 mg. P.O. once daily. Potassium 10 meq. P.O. once daily. 2 GM. sodium diet. 1. 2 gram sodium diet. 2. Metoprolol 25 mg. P.O. once daily. 3. Potassium 10 meq. P.O. once daily. 4. Diltiazem 120 mg. P.O. once daily.

3. Correct. This client is being treated for recurrent angina with hypertension. The admission prescriptions include spironolactone daily, which is a potassium-sparing diuretic. Therefore, the client should not be taking a daily dose of potassium. Prior to discharge, this client will also need instructions on avoiding additional potassium in the diet such as salt substitutes. 1. Incorrect. A "2-gram sodium diet" would be appropriate for a client with hypertension. 2. Incorrect. Metoprolol is a beta-blocker used to decrease preload, which will also decrease pulse and blood pressure. The dose is appropriate for this client and does not need to be questioned. 4. Incorrect. Diltiazem is a calcium channel blocker which vasodilates the arterial system and reduces recurrent angina by decreasing afterload. Additionally, calcium channel blockers help to decrease blood pressure. This medication and dose are appropriate for this client and would not need to be questioned.

A client has been admitted for exacerbation of ulcerative colitis with severe dehydration. What is the best indicator that this client has an actual fluid deficit? 1. Stool count of 10 episodes of diarrhea in 24 hours. 2. Weight increase of 2 kg and a 24 hour output of 1000 mL. 3. Admission weight of 74.3 kg and 2 days later a weight of 72 kg. 4. Daily intake of 2400 mL and an output of 1600 mL, plus diarrheal stools.

3. Correct: Any acute weight gain or loss is fluid. Weight is the best measurement for fluid loss or gain. Acute weight losses correspond to fluid volume deficits. This client has lost 2.3 kg over a 2 day period, indicating a fluid volume deficit (FVD). 1. Incorrect: Although 10 loose stools would result in fluid loss, the stool count of 10 episodes of diarrhea is an inaccurate measurement. The amount of fluid loss can vary depending on the amount of diarrhea, 10 "episodes" does not indicate how much fluid is lost.2. Incorrect: Weight gains indicate fluid volume retention and excess. This question asks about fluid volume deficit. Also, it does not take into account the client's intake. Only the output is considered, so output has less meaning without being compared to the intake. 4. Incorrect: Daily I&O is good information to have when assessing fluid status, but the diarrhea stools are an inaccurate measurement. The weight remains the best measurement for indicating a fluid deficit.

On the third postoperative day, a client develops a fever of 103.3ºF (39.6ºC) shivering and nausea. The primary healthcare provider writes these prescriptions. Which should the nurse do first? 1. Apply cooling blanket for fever. 2. Give ceftriaxone 1 gram IVPB stat. 3. Draw blood cultures. 4. Give promazine 50 mg po PRN for nausea.

3. Correct: Blood cultures MUST be drawn immediately to identify the causative bacteria. Once the organism is identified, the primary healthcare provider will order organism specific antibiotics. Always draw blood cultures before administering the antibiotic. If antibiotics are given before the blood cultures are drawn, the culture will be inaccurate, and the client cannot be treated appropriately. 1. Incorrect: Application of a cooling blanket is appropriate, but the key in this question is to "fix the problem" ASAP. To treat the infection, the blood cultures must be drawn ASAP and be done before starting the antibiotics. 2. Incorrect: Antibiotics are not given until the cultures have been drawn. Administering the antibiotic first would cause the culture to be inaccurate. 4. Incorrect: Preventing shivering is appropriate, but remember, always pick the answer that is most life-threatening. In this case, treating the bacteria as soon as possible is the priority answer. This requires the culture be obtained ASAP so the antibiotic therapy can be initiated.

The charge nurse is observing the work of an unlicensed assistive personnel (UAP). Which observation will require the nurse to intervene? 1. Placing soiled linen in a hazardous waste linen bag outside of the client's room. 2. Closing the door when exiting the room of a client diagnosed with tuberculosis (TB). 3. Going between client rooms wearing the same pair of gloves to collect I&O reports. 4. Cleaning a blood pressure cuff with a disinfectant.

3. Correct: Gloves should be removed and hands washed before leaving each client's room. Gloves quickly become contaminated and then become a potential vehicle for the transfer of organisms between clients.1. Incorrect: No intervention is needed because this is an appropriate action. Do not carry soiled linen down the hall to place in a receptacle. 2. Incorrect: No intervention is required because this is an appropriate action. Clients with tuberculosis (TB) need to be on airborne precautions in a negative pressure room with the door closed. 4. Incorrect: Equipment used against intact skin should be thoroughly cleaned with low level disinfectant between uses to reduce the load of microorganisms to a level that is not threatening to the next client. Therefore, no intervention is needed since the action is appropriate.

A client is admitted to the emergency department reporting abdominal discomfort and constipation lasting 3 days. Which abdominal assessment data would the nurse report to the primary healthcare provider? 1. Striae. 2. Borborygmi. 3. High-pitched bowel sounds. 4. Tympany noted on percussion.

3. Correct: High-pitched bowel sounds are indicative of an early bowel obstruction and hypoactive bowel sounds develop as obstruction worsens. The additional signs presented are also clues of a possible obstruction. 1. Incorrect: Striae on the abdomen may be a sign of past weight changes such as those seen with weight gain from pregnancy. These do not create abdominal discomfort nor constipation. 2. Incorrect: Borborygmi are normal, loud, rumbling sounds from gas movement through the intestines or from hunger. These are easily audible bowel sounds. These are not typically associated with constipation but may be present with diarrhea. 4. Incorrect: This is a normal finding in the abdomen. Tympany is usually present in most of the abdomen caused by air in the gut (a higher pitch than the lungs). Tympany would be minimal in this case, dependent upon the degree of constipation, which would lead to a dull sound upon percussion.

A low income family with children lives in an old, run-down apartment building situated close to a salvage yard in a poor neighborhood. Which area of assessment would be most important for the home health nurse? 1. Immunization status 2. School-related problems 3. Lead poisoning 4. Signs of child abuse

3. Correct: Lead may be found in the soil around rusted cars and can cause lead exposure. Old paint contains lead. Chips of paint may be consumed by young teething children. Old, run-down apartments may also have pipes which contain lead. Exposure to and consuming even small amounts of lead can be harmful. No safe lead level in children has been identified, and lead can affect nearly every system in the body. Mental and physical development can be negatively impacted by lead in the body. 1. Incorrect: Although the nurse does need to check immunizations, the hints in the stem indicate several problems that should direct the focus to lead poisoning, which is the priority. Immunization should be administered if the child is not on schedule, but consequences of lead poisoning is much more serious. 2. Incorrect: There was nothing in the stem indicating school problems. This would not take priority over lead exposure assessment.4. Incorrect: Although poverty and poor housing conditions have been identified as environmental factors for potential abuse, the stem of this question does not provide additional cues that would indicate abuse. Assessment for lead poisoning would be the priority in this situation based on the environmental issues identified.

The nurse is assisting an unlicensed assistive personnel (UAP) move an obese and dependent client toward the top of the bed. Which action is most important to prevent shearing forces on the skin? 1. Each person puts hands under the client and slides client toward the top of the bed. 2. Apply powder to the sheet before pulling client toward the top of the bed. 3. Place turn sheet under the client and use it to slide the client toward the top of bed. 4. Seek assistance of another person before pulling up in bed.

3. Correct: Placing a turn sheet under the client before moving will prevent friction and shearing forces which may lead to an abrasion or skin tear. Pressure ulcers are more likely to develop in tissues where shear force injury has occurred. 1. Incorrect: This will not prevent shearing forces on the skin and may result in scratches to the skin if the staff are wearing rings or other jewelry. The shearing force is created by gravity pushing down on the client's body, creating a resistance to movement. It creates a downward and forward pressure on tissues beneath the skin. 2. Incorrect: Using powder may actually irritate the skin as it may be abrasive when client's weight pushes against it and the bed. 4. Incorrect: Another person to assist in controlling the head or holding the lower legs would be helpful; however, using three to move up without protecting the skin would not be beneficial to the client. Shearing force injury to the skin could still result.

The nurse is performing morning care on a client on the medical unit. What should the nurse do after changing a client's bed linen? 1. Hold the linen close to the body while transporting it to the dirty utility room. 2. Wear a gown and gloves to transport the linen to the biohazard container. 3. Place the linen into a leak proof container sitting outside the room. 4. Place the linen in a pillow case and set it on the floor until client care is completed.

3. Correct: Soiled linen should be placed in a leak proof container for transport off the unit to the laundry. Make sure the linen bags are not overfilled which would prevent complete closure. 1. Incorrect: Linen should be held away from the body to prevent contamination of the nurse's clothes. The linens should be handled as little as possible to avoid possible contamination of air, surfaces and persons.2. Incorrect: Gloves should always be worn when handling soiled linen. A gown is not necessary. Soiled linen should be carried away from the body with minimal handling to prevent contamination.4. Incorrect: Soiled linen should not be placed on the floor. All linens should be handled and transported in a way that will minimize contamination and maintain a clean environment for the client, healthcare workers and visitors.

An elderly male, diagnosed with chronic renal failure and depression, lives alone. Which question should the home health nurse ask first when assessing this client? 1. Have you had suicidal thoughts in the past? 2. How are you feeling today? 3. Have you had thoughts of harming yourself? 4. Do you have guns in your hom

3. Correct: Suicide assessment should begin with direct questions about the presence of suicidal thinking. The nurse should recognize that elderly men are at higher risk for committing suicide, especially those with a history of depression, chronic illness and isolation. 1. Incorrect: This question should be asked, but only after determining if suicidal thinking is present. 2. Incorrect: This question could be an introductory question to establish rapport, but it is not direct enough to use in suicide assessment. 4. Incorrect: This question should be asked if the client is considering using gun as a method of suicide or if he has a history of suicide attempts with a gun.

Which assessment finding by a nurse would best indicate a positive Mantoux tuberculin skin test in a client? 1. Formation of a vesicle that is 4 mm in diameter 2. A sharply demarcated region of erythema of 10 mm 3. A central area of induration of 15 mm surrounded by erythema 4. A circle of blanched skin surrounding the injection site

3. Correct: The basis of reading the skin test is the presence or absence of induration, which is a hard, dense, raised formation. This is the area that is measured. Sometimes the site has erythema, a reddening of the skin that can also have swelling. The erythema should NOT be measured. Reactions to the skin test will vary. Measure only the induration. An induration of 15 mm or more is positive in persons with no known risk factors of TB. Reactions larger than 15 mm are unlikely to be due to previous BCG vaccination or exposure to environmental mycobacteria. 1. Incorrect: The basis of reading the skin test is the presence or absence of induration, which is a hard, dense, raised formation. This is the area that is measured. Sometimes the site has erythema, a reddening of the skin that can also have swelling. The erythema should NOT be measured. Reactions to the skin test will vary. For example, this is a very large reaction with blistering, swelling, and redness. Make sure to record blistering, even if no induration is present. Palpate this induration gently, as it may be painful. Measure only the induration. The vesicle may have a different underlying cause. 2. Incorrect: The basis of reading the skin test is the presence or absence of induration, which is a hard, dense, raised formation. This is the area that is measured. Sometimes the site has erythema, a reddening of the skin that can also have swelling. The erythema should NOT be measured. Doing so would result in a false positive test for the client. 4. Incorrect: The basis of reading the skin test is the presence or absence of induration, which is a hard, dense, raised formation. This is the area that is measured. Sometimes the site has erythema, a reddening of the skin that can also have swelling. The erythema should NOT be measured. Reactions to the skin test will vary. The area around the injection site may appear blanched initially, but should resolve. However, blanching should not be measured.

An angry client visits the primary healthcare provider's office and requests a copy of their medical records. The client is angry after being placed on hold several times for over 10 minutes when requesting an appointment. What should the nurse tell this client? 1. All client appointment calls are transferred to the scheduling clerk. 2. The client will have to speak to the primary healthcare provider. 3. A copy of the record may be obtained within 24 hours of the request. 4. Medical records must stay within the facility unless requested by another primary healthcare provider.

3. Correct: The client has the right to the personal medical record. Generally, a period of time is required to get the record copied. The client may be charged for the copy. This assures the client that the request will receive attention. 1. Incorrect: This response dismisses the client's feelings and may only anger the client further. The response does not address the reason for the client's anger. The statement may be true; however, the client does have the right to request and receive a copy of the medical record. 2. Incorrect: The primary healthcare provider does not have to be contacted, as there should be policies in place to grant the request for a copy of the medical record. Also, telling the client to speak to the healthcare provider would not address the reason for the client's anger. This would dismiss the client's feelings. 4. Incorrect: The client has a right to the medical record. Records may also be requested by other providers with consent of the client. The client's feelings should be addressed and the client should be informed that the medical record will be provided as requested.

A client was admitted to the psychiatric unit with delusions and a history of auditory hallucinations. The client reports, "The FBI has been watching my house and are going to raid it and arrest me." What is the nurse's best response? 1. The FBI would not be watching you unless there was a good reason. 2. I don't think that the FBI is watching your house. 3. I believe that your thoughts are very disturbing to you. 4. Tell me more about your thoughts.

3. Correct: The client's delusions can be very distressing. The nurse should empathize with the feelings of the client, but should not validate the belief itself. Empathy displays that the nurse is concerned, interested, and accepts the client but does not support the delusion. 1. Incorrect: Arguing with the client who has delusions only upsets the client and may provoke violence. The client can not understand the logical argument, so the delusional ideas are not dispelled. Also, the argument can interfere with the development of trust. 2. Incorrect: Disagreement may anger the client. The client needs empathy and understanding from the nurse. This is dismissing the client's feelings. The focus should not be on what the nurse thinks. The focus should always be on the client's feelings. 4. Incorrect: In-depth detail of delusions only reinforces the delusion. The nurse should encourage reality based conversation. Interacting about reality is beneficial for the client to move them away from delusional thoughts.

A nurse from an adult unit was reassigned to the pediatric unit. Which client would be least appropriate to assign to this nurse? 1. Ten year old with 2nd and 3rd degree burns. 2. Five year old that was in a MVA and has a femur fracture. 3. Six year old admitted for evaluation of possible sexual abuse by a parent 4. Two month old with bronchopulmonary dysplasia being admitted for reflux.

3. Correct: The least appropriate client to assign the nurse from the adult unit would be the suspected sexual abuse. Caring for an abused child requires skill that must be developed from understanding the dynamics of abuse as well as working with a certain developmental level. 1. Incorrect: A nurse on an adult unit should understand classification of burns and associated care for the burn client. The pediatric burned client would be a similar to the condition adults might acquire, and the nurse's skill level could transfer to these clients. 2. Incorrect: The nurse who works on an adult unit should understand the concepts for caring for a client with a fracture. The pediatric client with the fracture would be a similar condition adults might acquire, and the nurse's skill level could transfer to these clients. 4. Incorrect: The 2 month old with BPD is different, but the concept and care of reflux is similar to that in adult clients.

What assignment would be most appropriate for the nurse to delegate to the unlicensed assistive personnel (UAP)? 1. Teaching the client perineal care. 2. Changing a colostomy bag on a client. 3. Serving the diet tray for a diabetic client. 4. Taking the initial vital signs on a client who is to receive blood.

3. Correct: The most appropriate task for a non-licensed person would be serving the diet tray for a client. This does not require experience for a particular skill nor does it require higher level skills that would require a licensed person to perform. 1. Incorrect: Teaching is the responsibility of the RN and cannot be delegated to a LPN nor a non-licensed personnel. 2. Incorrect: Changing the colostomy bag on a client will need someone with the experience/skill of performing this task. Although some agencies allow UAP's to change colostomy bags, there may be further assessment needed associated with the ostomy, such as skin condition around the ostomy. This would not be the best option to assign to the UAP. 4. Incorrect: UAPs can take VS, but they must be very cautious in order to note changes and the client receiving blood should be assessed for any s/s of reaction. Therefore, it would be best for the licensed personnel to obtain the initial v/s prior to blood administration to assess the client's status and have a baseline for evaluating the client's response to the blood administration.

A 70 year old client was admitted to the vascular surgery unit during the night shift with chronic hypertension. At 0830, the unlicensed nursing assistant (UAP) reports that the client's BP is 198/94. What would be the best action for the charge nurse to delegate at this time? 1. Ask the UAP to put the client back in bed immediately. 2. Tell the UAP to take the BP in the opposite arm in 15 minutes. 3. Have the LPN/LVN administer the 0900 furosemide and enalapril now. 4. Ask the LPN/LVN to assess the client for pain.

3. Correct: The nurse should recognize the need for measures to reduce the blood pressure. Administering the client's blood pressure medicine is aimed at correcting the problem. It is appropriate to administer the medications at this time in relation to the time that the next dose is due. 1. Incorrect: This is an appropriate action, but does not address the problem of lowering the client's blood pressure. 2. Incorrect: This is an appropriate action, but does not address the problem of lowering the client's blood pressure. 4. Incorrect: This is an appropriate action, but does not address the problem of lowering the client's blood pressure.

The triage nurse in the emergency department (ED) assesses 4 clients. Which client is in need of emergent care? 1. A 52 year old who has a partially amputated finger. 2. A 9 month old with temperature of 103°F (39.4°C). 3. A two year old with excessive drooling and a weak cough. 4. A 28 year old experiencing a migraine headache for three days.

3. Correct: The two year old is exhibiting signs of respiratory difficulty with excessive drooling and a weak cough. Partial airway obstruction is likely and maybe the result of acute epiglottitis in which rapid progression to severe respiratory distress can occur . Airway takes priority over the other clients.1. Incorrect: The partial amputation would have associated bleeding could be seen next, but airway takes priority. 2. Incorrect: Most fevers in children do not last for long periods and do not have much consequence. Elevated temperature would not take priority over airway. Antipyretics can be given in triage. 4. Incorrect: The migraine is not emergent. Take care of life-threatening illnesses/injuries first. Remember, pain never killed anyone.

The nurse is teaching a client about foods containing tyramine which should be avoided while taking a monoamine oxidase inhibitor (MAOI). Which meal selection by the client would indicate understanding of an acceptable food to eat? 1. Smoked turkey and dressing, sweet peas and carrots and milk. 2. Baked chicken over pasta with parmesan sauce, baked potato and tea. 3. Fried catfish, French fries, coleslaw and apple juice. 4. Liver smothered in gravy and onions, rice, squash and water.

3. Correct: These foods are not high in tyramine. Tyramine is an amino acid that helps in the regulation of blood pressure. MAOIs block the enzyme monoamine oxidase which is responsible for breaking down excess tyramine in the body. Eating foods high in tyramine while on MAOIs can result in dangerously high levels of tyramine in the body. This can lead to a serious rise in blood pressure, creating an emergency situation. Tyramine is found in protein-containing foods and the levels increase as these foods age. Food such as strong or aged cheese, cured meats, smoked or process meats, liver (especially aged liver), pickled or fermented foods, sauces, soybeans, dried or overripe fruits, meat tenderizers, brewer's yeast, alcoholic beverages and caffeine- such as in tea, cokes and coffee are considered to be high in tyramine and should be avoided in clients taking MAOIs. 1. Incorrect: The following foods in the options listed above contain moderate to high levels of tyramine and should be avoided while taking MAOIs: smoked turkey, parmesan cheese, tea and liver.2. Incorrect: The following foods in the options listed above contain moderate to high levels of tyramine and should be avoided while taking MAOIs: smoked turkey, parmesan cheese, tea and liver.4. Incorrect: The following foods in the options listed above contain moderate to high levels of tyramine and should be avoided while taking MAOIs: smoked turkey, parmesan cheese, tea and liver.

The nurse is preparing a client for a renal biopsy. Which is most important for the nurse to assess prior to this procedure? 1. BUN and creatinine 2. NPO status and signature on consent 3. Bleeding time and coagulation studies 4. Serum potassium and urine sodium

3. Correct: Yes. Before you insert a needle into an organ for a biopsy, it would be best to know the client's bleeding time because there is a risk of bleeding when the biopsy is performed. 1. Incorrect: Although these are related to renal function, they do not impact the procedure itself. Therefore, they are not essential for the procedure. 2. Incorrect: Although both of these are carried out, they are not the priority over risk of bleeding. Always think what could be life threatening. 4. Incorrect: Although both serum potassium and urine sodium are related to renal function, they do not impact the procedure itself. Therefore, they are not essential for the procedure.

Which clients would be appropriate for the RN to assign to an LPN/LVN? 1. Seventy four year old client with unstable angina who needs teaching for a scheduled cardiac catheterization. 2. Sixty year old client experiencing chest pain scheduled for a graded exercise test. 3. Forty eight year old client who is five days post right-sided cerebral vascular accident (CVA). 4. Eighty four year old client with heart disease and mild dementia. 5. Newly admitted ninety year old client with decreased urinary output, altered level of consciousness, and temperature of 100.8°F (38.2°C) 6. Sixty six year old client with chronic emphysema experiencing mild shortness of breath.

3., 4., 6. Correct: The client who is five days post CVA is one of the most stable clients and could be assigned to the LPN/LVN. There is nothing in the option to indicate that this client is unstable. There is no indication that the eighty-four year old client with heart disease and dementia is unstable so this client can be assigned to the LPN/LVN. The client with chronic emphysema will experience shortness of breath. There is nothing to indicate that this client is unstable. 1. Incorrect: This client is unstable and should be cared for by the RN. Additionally, the RN is responsible for teaching. 2. Incorrect: This client is experiencing chest pain and is thus considered unstable and should be cared for by the RN. 5. Incorrect: This client has s/s that could indicate sepsis, so is considered unstable and should not be assigned to the LPN/LVN.

A nurse is at highest risk for blood-borne exposure during which situation? 1. When removing a needle from the syringe. 2. While placing a suture needle into the self-locking foreceps. 3. Prior to inserting the intravenous (IV) line, the client moves causing a needle stick to the nurse. 4. A clean needle sticks the nurse through blood-soiled gloves.

4. Correct: A clean needle that moves through blood-soiled gloves to stick the nurse is considered to be potentially contaminated and results in a blood-borne exposure. All other answers are considered a clean stick.1. Incorrect: This is considered a clean stick. The needle is sterile initially and has not been contaminated prior to removal of the needle from the syringe. 2. Incorrect: This is considered a clean stick since the suture needle has not been inserted into the client prior to the needle stick. 3. Incorrect: This is considered a clean stick. The IV insertion device is sterile and has not been contaminated since it was not inserted into the client.

Which client admitted to the emergency department should the nurse assess first following shift report on assigned clients? 1. Client reporting inability to void and a distended bladder on palpation. 2. Client diagnosed with a confirmed closed fracture of the tibia. 3. Client who has a suspected corneal laceration. 4. Client with abdominal discomfort and a rigid abdomen on palpation.

4. Correct. A rigid abdomen may indicate bleeding or other causes of peritonitis which takes priority over the other three, more stable clients. This could lead to shock in this client. Conditions requiring immediate treatment include cardiac arrest, anaphylaxis, multiple trauma, shock, poisoning, active labor, drug overdose, severe head trauma, and severe respiratory distress. 1. Incorrect. Although this condition may be uncomfortable and could lead to renal problems if not resolved, it does not take priority over a client who is bleeding. 2. Incorrect. This person is likely experiencing pain, but this client does not take priority over a client who has peritonitis and may be going into shock from bleeding or third spacing into the peritoneum. Remember, pain never killed anyone. 3. Incorrect. This client with a corneal laceration would be experiencing pain and needs attention to avoid vision loss. However, this client does not take priority over a client who has peritonitis and may be going into shock from bleeding or third spacing in the peritoneum. Remember, ascites is fluid in the peritoneal cavity.

A client has sustained a major head injury as a result of a motor vehicle accident. The emergency department nurse is assessing the client's neurological status every 15 minutes. Which sign would the nurse recognize as an early indicator of an increased intracranial pressure (ICP)? 1. Dilated and unresponsive pupils 2. Cheyne-Stokes respirations 3. Cushing's triad 4. Change in level of consciousness (LOC)

4. Correct: A change in LOC is one of the earliest indicators of an elevated ICP. 1. Incorrect: Loss of papillary reflexes is a late sign of increased ICP. Earlier pupil changes would include gradual dilation and pupils become sluggish in response to light. 2. Incorrect: This is a late sign of increased ICP. This pattern of respirations is characterized by an increase in depth and rate of respirations followed by a gradual reduction. 3. Incorrect: Cushing's triad is a very late presentation of brain stem dysfunction and manifest as bradycardia, hypertension, and bradypnea. It is seen when cerebral blood flow decreases significantly. This is a grave sign for a client with a head injury. It is related to a significant increase in ICP. Therefore, it is not one of the earliest indicators of an elevated ICP. It is a late sign and if intervention is not initiated, herniation of the brain stem is imminent, with death likely.

After reviewing the nursing notes on a client receiving a unit of packed red blood cells, what action should the charge nurse take? 1. Decrease the transfusion rate to 50 mL/hour. 2. Assess the client for a transfusion reaction. 3. Check primary healthcare provider prescription for prescribed administration time. 4. Stop the transfusion and send blood bag to the lab.

4. Correct: All blood from each unit of packed red blood cells must be completed within a 4 hour time frame due to risk of hemolysis and bacterial invasion. If the unit of blood is not completed in a 4 hour time frame, the blood must be sent to the lab to be discarded. Keep in mind that the time frame for administering platelets and fresh frozen plasma differs (20-30 min). 1. Incorrect: This blood has been hanging for 4 hours and must be discontinued. 2. Incorrect: The problem is that the blood has been hanging too long. It must be taken down. There is no indication that a transfusion reaction is occurring. Transfusion reaction symptoms include back pain, dark urine, chills, fainting or dizziness, fever, flank pain, skin flushing, shortness of breath. 3. Incorrect: The problem is that the blood has been hanging too long. It must be taken down. It cannot be hung for a longer period of time due to risk of hemolysis and bacterial invasion.

How would the nurse determine the correct size oropharyngeal airway for a client? 1. Select the same size as the little finger of the victim. 2. Measure from the tip of the lips to the epiglottis. 3. Determine the length from the earlobe to the xiphoid process. 4. Measure from the earlobe to the corner of the mouth.

4. Correct: An airway of proper size will extend from the corner of the client's mouth to the tip of the earlobe on the same side of the client's face. 1. Incorrect: The size of the client's little finger does not determine the size of the oral airway that should be used. This would result in an inappropriate size oropharyngeal airway to be selected. 2. Incorrect: The epiglottis is an internal body part thus making it impossible to correctly measure it. In addition, the measurement would not determine the appropriate size oropharyngeal airway to use. 3. Incorrect: Measuring from the client's earlobe to the client's xiphoid process would make the oral airway too long.

What medication should the nurse anticipate giving to a client in preterm labor to stimulate maturation of the baby's lungs? 1. Magnesium sulfate 2. Terbutaline 3. Methotrexate 4. Betamethasone

4. Correct: Betamethasone is used to stimulate maturation of the baby's lungs in case preterm birth occurs. This medication is given to help prevent respiratory distress syndrome (RDS) by improving storage and secretion of surfactant that helps to keep the alveoli from collapsing. 1. Incorrect: Magnesium sulfate is given to stop preterm labor, however, if delivery is imminent, then Betamethasone should be given to stimulate maturation of the baby's lungs. 2. Incorrect: Terbutaline is contraindicated in preterm labor, however, if delivery is imminent, then Betamethasone should be given to stimulate maturation of the baby's lungs. 3. Incorrect: Methotrexate is used to stop the growth of the embryo in ectopic pregnancy so that the fallopian tube can be saved. It is not an agent used in the management of preterm labor.

A female client taking captopril for hypertension tells the clinic nurse that she is planning to get pregnant. What recommendation should the nurse make? 1. "Captopril can be taken safely during pregnancy, but we will need to decrease your dose so you do not become hypotensive." 2. "We will need to increase your dose of captopril once you become pregnant." 3. "In order to prevent neural tube defects, start taking folic acid." 4. "Captopril can cause serious harm to an unborn baby, so you must prevent pregnancy while taking this medication. "

4. Correct: Captopril should not be taken during pregnancy because serious harm (possibly fatal) to the unborn baby can result when taken during pregnancy.1. Incorrect: Captopril should not be taken during pregnancy because serious harm (possibly fatal) to the unborn baby can result when taken during pregnancy. 2. Incorrect: Captopril should not be taken during pregnancy because serious harm (possibly fatal) to the unborn baby can result when taken during pregnancy. 3. Incorrect: Captopril should not be taken during pregnancy because serious harm (possibly fatal) to the unborn baby can result when taken during pregnancy. The problem being presented in the stem is not related to general prevention of neural tube defects. Folic acid would not prevent the harm to the fetus caused by catopril.

A client in a psychiatric unit sings over and over, "It is hot, I am a hot tot in a lot, I sit all day on a cot drinking a pop." How should the nurse document this form of thought? 1. Neologisms 2. Dissociation 3. Fugue 4. Clang Association

4. Correct: Clang association involves the choice of words governed by sounds, often taking the form of rhyming even though the words themselves don't have any logical reason to be grouped together. 1. Incorrect: The psychotic person invents new words, or neologisms, that are meaningless to others but have symbolic meaning to the psychotic person. 2. Incorrect: Dissociation is the splitting off of clusters of mental contents from conscious awareness. It is a mental process that leads to a lack of connection in the client's thoughts, memory and sense of identity. In its mild form, it is similar to day dreaming. In a more severe form, it can be manifested as multiple personalities. 3. Incorrect: Fugue is sudden, unexpected travel away from home or customary place of daily activities, with inability to recall some or all of one's past. The person is unaware that anything has been forgotten. Following recovery, there is no memory of the time during the fugue.

Which snack selection by a client receiving chemotherapy would indicate to the nurse that teaching has been successful? 1. Fresh salad with cucumbers, carrots, and tomatoes. 2. Orange slices with yogurt. 3. Strawberries with whipped cream. 4. Milk shake with a packet of instant breakfast added.

4. Correct: Clients with cancer often experience a combination of increased energy expenditure but the nutritional intake is decreased and inadequate to meet the caloric and protein needs. The decreased intake may be, in part, due to the side effects of the chemo. Cold drinks, like shakes, can be soothing, especially if the client has no desire to eat solid foods or is experiencing mouth pain. Shakes will also offer more calories for the client and more protein if a packet of instant breakfast or protein powder is added. Cold, high protein foods are generally tolerated better and have less offensive odors than hot foods. 1. Incorrect: Although raw vegetables are full of fiber, vitamins and minerals, this is not the best option because they can contribute to food borne illness, especially in a person who is immunosupressed from receiving chemotherapy. Their immune system is weakened. To minimize the risk of infection, vegetables should be cooked for these clients. It is recommended that all fruits and vegetables be washed thoroughly in cold water and dried on a clean paper towel. 2. Incorrect: Oranges, grapefruits, and tangerines should be avoided in favor of softer, blander fruits such as bananas. Mouth sores are a common side effect of chemo treatment which can be extremely painful when even the blandest food rubs up against the sore. The citrus nature of oranges can cause discomfort if stomatitis is present. 3. Incorrect: Raw fruit with rough texture such as strawberries and raspberries should not be eaten. The rough texture on the strawberries can hide dirt and other contaminants that washing can not clean well enough.

During a conversation with a client on a psychiatric unit the client tells the nurse, "Everyone here hates me." Which response by the nurse is best? 1. No, they do not hate you. 2. What did you do to make others not like you? 3. Just don't pay attention to what others think of you. 4. I can't speak for the other people, but I don't hate you.

4. Correct: Here the nurse is speaking only for the nurse. The nurse cannot legitimately speak for anyone else. The nurse must model the process of not speaking for anyone else. The response also lets the client know that the nurse cares about the way the client feels. 1. Incorrect: This is arguing and defending which are non-therapeutic communication techniques. The nurse does not know how the others on the unit feel about the client, so this may not be a true statement. Arguing with a client's belief can further upset or anger the client and leads to mistrust of the nurse. 2. Incorrect: This is agreeing with the client that everyone hates the client. It also puts the client on the defense by implying that the client is at fault for doing something that made everyone hate the client. This response reinforces the client's false belief. 3. Incorrect: This is using denial. This is where the nurse denies that a problem exists and blocks the discussion with the client. This avoids helping the client identify and explore the problem. This also dismisses the client's feelings.

A client comes into the emergency department (ED) and demands to be seen immediately, but refuses to tell the triage nurse the problem. During the assessment, the client starts yelling and shaking their fist. For the nurse's safety, what should be the nurse's initial action? 1. Tell the client to stay calm, and that treatment will be provided soon. 2. Explain that unless the client behaves, they will be sent away from the ED. 3. Notify the client that security will be called if they do not go to the waiting room immediately. 4. Find a safe place away from the client and then notify security.

4. Correct: Self-protection is a priority. There is no advantage to protecting others if medical caregivers are injured. Security officers and police must gain control of the situation first, and then care is provided.1. Incorrect: This does not provide safety for the nurse and might increase the client's anger. 2. Incorrect: This is not a true statement and does not provide immediate safety for the nurse. Clients seeking treatment are not refused care in the ED.3. Incorrect: This is not the initial action. Finding a safe place is the first action for the nurse's safety. Also, the angry client does not need to be sent to the waiting room around other clients at this time.

The nurse should question which prescription for a client diagnosed with acute heart failure? 1. 2 gram of sodium (Na) diet. 2. Digoxin 0.25 mg IV q 4 hours times 3 doses. 3. Furosemide 40 mg IVP stat. 4. Start IV with NS at 125 mL/hr.

4. Correct: The client is in fluid overload and does not need the normal saline (NS) at 125 mL/hr. NS is an isotonic solution. It goes in the vascular space and stays there without shifting out to the cells. This could cause additional overload in the vascular space as well as cause the BP to increase. The other prescriptions are acceptable. 1. Incorrect: This is an appropriate measure Na restricted diet will help to lower the serum Na and decrease H2O retention. This does not need questioning. 2. Incorrect: Digoxin is a digitalis glycoside. It slows conduction and strengthens the force of contraction of the heart. Therefore, this medication that increases cardiac contractility and reduces the heart rate does not need questioning. 3. Incorrect: Furosemide is a diuretic. It enhances renal excretion of Na and H2O and reduces systemic and pulmonary congestion. This medication prescription does not need questioning.

When preparing a client for surgery, the nurse realizes the operative permit has not been signed. The client tells the nurse he understands the procedure, but received his preoperative medication approximately 10 minutes ago. What would be the appropriate action by the nurse? 1. Have the client sign the permit, as he verbalizes understanding. 2. Witness the form after having the client sign it. 3. Have his wife sign the form as she witnessed his statement that he understands. 4. Call the surgical area and explain that the surgery will have to be cancelled.

4. Correct: The client must sign the operative permit or any other legal document prior to taking preoperative drugs that can affect judgment and decision-making capacity. 1. Incorrect: The client's verbal understanding does not override the fact that he has received medication that can alter thought processes and decision-making. 2. Incorrect: Witnessing would not make this document legal. The consent would not be valid because the client has already received the pain medication that could alter the thought process. 3. Incorrect: When a client is of legal age (unless an emancipated minor) and of sound mind, it would be inappropriate for the spouse to sign the form for surgery. In order to be valid it must be the client who signs it, unless there is a legal power of attorney, durable power of attorney, or healthcare surrogate.

The nursing supervisor notified the charge nurse on a pediatric unit that a child with a history of developmental delays is being admitted with shingles. The nurses on the floor have the following assignments. It would be inappropriate for the charge nurse to assign the new admit to which nurse? 1. A nurse caring for clients with nephritis, irritable bowel syndrome, and appendectomy. 2. A new nurse just out of orientation caring for clients diagnosed with RSV, asthma, and anorexia nervosa. 3. A nurse caring for clients diagnosed with spina bifida, Hirschsprung's Disease, and irritable bowel syndrome. 4. A pregnant nurse caring for clients with cystic fibrosis, myelomeningocele, and rheumatoid arthritis.

4. Correct: The information does not let you know if any of the nurses have had chickenpox or not. If a nurse has not had chickenpox, then they should not care for the client with shingles. The varicella zoster virus is responsible for chickenpox and shingles. The virus is lying dormant in the nerve ganglia and under certain conditions erupts (for example: stress). With the information you have, it would be best not to assign the new admit to the nurse who is pregnant. The other set of nurses and clients have no identified contraindications to taking care of the client with shingles. 1. Incorrect: This is an appropriate assignment. There are no identified contraindications for the nurse or clients to prevent the nurse from caring for a client with shingles. 2. Incorrect: This is an appropriate assignment. There are no identified contraindications for the nurse or clients to prevent the nurse from caring for a client with shingles. 3. Incorrect: This is an appropriate assignment. There are no identified contraindications for the nurse or clients to prevent the nurse from caring for a client with shingles.

A client had an abnormal maternal serum alfa fetoprotein (MSAFP) at 18 weeks gestation. She is now 22 weeks gestation, and an amniocentesis has just been completed for genetic analysis. Which nursing action has priority? 1. Monitor the needle entry site for signs of infection. 2. Encourage the client to express her feelings. 3. Assess the maternal blood pressure for hypertension. 4. Monitor fetal heart tones and uterine activity.

4. Correct: There is a risk for pregnancy loss after amniocentesis. The priority nursing intervention is to monitor for fetal heart tones and uterine contractions. 1. Incorrect: There would not be signs of infection so soon after the procedure. 2. Incorrect: The physiological needs have priority over psychological/self-esteem needs (Maslow's hierarchy). The fetal status should be the priority to be monitored. The client's feelings are important and would be assessed, but would not take priority over the fetal status. 3. Incorrect: Hypotension is a more likely side-effect of the procedure. Since the procedure involved entering the amniotic sac, fetal status should be a priority assessment.

The nurse is preparing to administer nadolol to a hospitalized client. Which client data would indicate to the nurse that the medication should be held and the primary healthcare provider notified? 1. Blood pressure 102/68 2. Glucose 118 3. UOP 440 mL over previous 8 hour shift. 4. Heart rate 56/min

4. Correct: This is a beta blocker. It slows the heart rate. If a client's heart rate is less than 60 beats per minute, notify the primary healthcare provider and ask if the client should receive this medication. Administering a beta blocker to a client who has a heart rate less than 60 could possibly cause the client to develop symptomatic bradycardia and hypotension. 1. Incorrect: If the client's BP drops below 90/60, this beta blocker should be held and the primary healthcare provider notified. The BP in this option is high enough to administer the medication, but the BP in clients on beta blockers should be monitored and the client should be taught about signs and symptoms of hypotension. 2. Incorrect: This is a normal glucose level. If the client is a diabetic, beta blockers can mask the signs of hypoglycemia. There diabetics on beta blockers should monitor their blood sugar carefully. 3. Incorrect: Urinary output is adequate. Beta blockers do not alter renal function. However, if pulse and BP are reduced too much, renal perfusion could ultimately be affected.

A client with cancer of the larynx undergoes radiation therapy for 5 weeks prior to a neck dissection and tumor excision. The client asks the nurse how long the post surgical recovery time will be. How should the nurse reply? 1. "I really don't know. It is different for everyone, but speak to your surgeon." 2. "Your medical insurance will cover the whole length of your stay, so don't worry." 3. "You shouldn't worry about how long you are going to stay. You should focus on getting better." 4. "It may be a little longer than average. The radiation you received sometimes delays tissue healing."

4. Correct: This is the best, most accurate response. Radiation can cause tissue trauma and changes that can delay wound healing. 1. Incorrect: On NCLEX®, the nurse should know not to put work off on someone else. This answer avoids responsibility and does not provide the client with the information requested. 2. Incorrect: This answer assumes the client has financial concerns, but this is not the question the client asked. It also dismisses the client by being told not to worry. 3. Incorrect: This answer brushes off the client. Never pick an answer that brushes off the client's concern.

A client diagnosed with a duodenal ulcer is prescribed lansoprazole and sucralfate. What should the nurse teach the client about how to take these medications? 1. Take together immediately before meals. 2. Take together immediately after meals. 3. Take the sucralfate first, wait at least 30 minutes, then take the lansoprazole. 4. Take the lansoprazole first, wait at least 30 minutes, then take the sucralfate.

4. Correct: When prescribed any medication along with sucralfate, the client should avoid taking the medication at the same time with sucralfate. Sucralfate can make it harder for the body to absorb lansoprazole because it forms a "coating" or "barrier" on the stomach lining. Therefore, the client should wait at least 30 minutes after taking the lansoprazole before taking sucralfate. 1. Incorrect: Taking sucralfate and lansoprazole at the same time will decrease the effects of lansoprazole because the sucralfate coats the stomach lining and reduces the absorption of the lansoprazole. 2. Incorrect: Taking sucralfate and lansoprazole at the same time will decrease the effects of lansoprazole because the sucralfate coats the stomach lining and reduces the absorption of the lansoprazole.. 3. Incorrect: Sucralfate can make it harder for your body to absorb lansoprazole because of the barrier created on the stomach lining.

The primary healthcare provider has prescribed phenytoin 100 mg intravenous push (IVP) stat through a non-tunneled central venous catheter lumen with no other medication or fluid infusing. In what order should the nurse administer this prescription? Cleanse access port Connect 10 mL normal saline to access port Gently aspirate for blood Flush saline using push-pause method Administer phenytoin Flush with normal saline, then with heparin

Proper administration of medication through a non-tunneled central venous catheter: First, cleanse the access port. Failure to cleanse the port first would increase the risk of infection from contamination when the port is accessed. Second, connect 10 mL normal saline to access port. This 10 mL syringe will be connected to first check patency and then for flushing prior to medication administration. At least 10 mL of normal saline is used to flush central lines. Third, gently aspirate for blood. Fourth, flush saline using push-pause method. This method is utilized to help clear the catheter of blood or drugs that could potentially adhere to the internal surface of the central line catheter. This creation of turbulent flow from pausing then pushing causes swirling of the fluid and theoretically removes blood and medications from the walls of the catheter, which reduces the risk of occlusion in the catheter. Fifth, administer phenytoin. Sixth, flush with normal saline, then with heparin. Standard flushing solutions used most frequently for central venous access devices include normal saline and/or heparinized sodium chloride. Low dose heparin flushes are generally used to fill the lumen of the central line between use in order to prevent thrombus formation and maintain patency of the catheter for a longer period of time.

A healthy newborn has just been delivered and placed in the care of the nurse. What nursing actions should the nurse initiate? Place in the correct priority order. Assess newborn's airway and breathing. Bulb suction excessive mucus. Assess newborn's heart rate. Place identification bands on newborn and mom. Administer sterile ophthalmic ointment containing 0.5% erythromycin.

Remember Maslow's hierarchy of needs will guide your assessment. First, Assess newborn's airway and breathing. The most critical change that a newborn must make physiologically is the initiation of breathing. The nurse should assess the newborn's crying. If the cry is weak, it may indicate a respiratory disturbance. Other signs of respiratory compromise may include: stridor, grunting, retractions, apnea or diminished breath sounds. Normal respiration are 30 - 60 breaths a minute. Second, Bulb suction excessive mucus. It is important to assure that the throat and nose are kept clean of secretions to prevent respiratory distress. Third, Assess newborn's heart rate. If there is no respiratory distress, the nurse continues the assessment by checking the heart rate and other vital signs. Fourth, Place identification bands on newborn and mom. These are critical for ensuring babies and moms will be appropriately matched at all times but does not take priority over respiration and circulation. Fifth, Administer sterile ophthalmic ointment containing 0.5% erythromycin. This is a legally required prophylactic eye treatment to prevent Neisseria gonorrhea. However, this would never be a priority over Maslow's hierarchy of needs.


Ensembles d'études connexes

Structuralism & poststructuralism

View Set

Chapter 14 Health Care Providers

View Set

CSI CHAPTER 2 (BINARY VALUES AND NUMBER SYSTEM)

View Set